You are on page 1of 40

The Practice, Process & Politics of Pain Management

number of persons
Volume XIV, Number 2 From the accept that medication

Inside this issue President and treatments alleviate


pain, skepticism among
From the Editor, 2 Karsten F. Konerding, MD
Duncan C. MacIvor, MD lay persons and physi-
Pain - Politics and Public cians persists. Some
Perception: Virginias Why Pain? people doubt the credi-
Experience, 4
Stephen P. Long, MD Thanks to Wilhelm Conrad Roentgen, a bility of the chronic
Modern Treatment of physics professor at the University of pain sufferer, others doubt the motives of a too
Chronic Pain, 8 Wurzburg, the world is accustomed to seeing eager prescriber, many people fear the addic-
John M. Barsanti, MD
radiographs of skeletons, hands, feet, lungs tive nature of drugs, and many physicians fear
Chronic Pain from a Chemical
and other major organs. Little did Roentgen the legal snares of under or overprescribing.
Dependence Perspective, 11
Peter R. Coleman, MD know in 1895, however, that his x-ray would Yet, chronic pain exists.
The Pharmacology of Opioids, 13 advance medicine immeasurably and provide Why Pain? Why a special edition of our
Drs. Michael Weaver, Deborah Haller,
Martha J. Wunsch
physicians with an invaluable tool in diagnos- newsletter on pain management? The man-
The Diagnosis and Treatment
ing illness and disease. agement of pain is on the cusp of a new hori-
of Headache, 16 zon in medicine. As physicians, regardless of
Alan R. Towne, MD
Not surprisingly, the world was slow to appre-
ciate what Professor Roentgen offered. One our specialty, we cannot afford to delegate
Trigeminal Neuralgia, 23
Robert F. Saul, MD New York paper reported that x-rays would be public discussion and policymaking decisions
Distinguishing Between Chronic used to help medical students learn by reflect- about pain management to others, for tomor-
Pain and Drug-Seeking Behavior ing anatomic diagrams directly into their row we may wake up and do battle with
in Pain Management/Opioid
brains. A London newspaper referred to the certified pain specialists who may not
Abuse, 25
Michael Weaver, MD invention as a revolting indecency, and cloth- be physicians.
Chronic Pain Patient or Drug ing manufacturers quickly promoted Why Pain? Because medical literature in-
Seeker, 28
Special Agent Carlton Johnson, Jr.
x-rayproof garments for women. forms us that an estimated 80 million Ameri-
Listening to the Story of Suffering: Fortunately, the merits of x-ray slowly became cans have some form of chronic pain, mostly
Psychiatric Perspectives of Pain of noncancerous origin. A significant number
Diagnosis and Management, 29
known through medical education and public
James E. Sellman, MD education. Within decades, most people came of these people have pain that does not respond
The Legal Issues: How the to see that, with radiographic methods, injuries to commonly used surgical and medical treat-
Courts See Pain Management, 32 to bones otherwise covered by flesh and tu- ments. Too many of these patients endure
Stephen D. Rosenthal, Esquire
mors hidden deep in the chest could be seen, chronic pain for too long, simply because too
Issues Regarding Licensure, 34
treated and, perhaps, cured. Today, the value of many physicians do not know how to diag-
William L. Harp, MD
the diagnostic radiologistthe eye of medi- nose, refer or treat chronic pain.
Alternative Therapy Approaches:
How Acupuncture Can Be cineis little disputed. Why Pain? Persistent pain is not simply a be-
Effectively Used, 36
M. Craig Pinsker, MD, PhD In many ways, pain management today is nign, unpleasant symptom of something that
The Role of the Physiatrist in where radiology was in its infancy. We are just cannot be fixed. A growing body of scientific
Managing Chronic Nonmalignant evidence and literature supports the clinical
Pain, 38
beginning to understand and appreciate the
Jane Pendleton Wootton, MD complexity of the pain problem and to realize impression that prolonged pain is aggressive,
the varieties of treatments available. The world physically destructive, and an emotionally ex-
knows that pain exists, just as it knew that hausting disease. Pain can result in measurably
bones lay beneath flesh, and while a growing harmful changes in hormonal and metabolic
Continued on page 2

May 2002
R A M I F I C AT I O N S

From the President


Continued from page 1
From the Editor
Duncan MacIvor, MD

functions, alterations of brain chemicals An Imperfect World


vital to pain modulation, suppression of
immune system responses, and interfer- Nothing in medicine is more fundamental trouble. What is a
ence with gene expression. These are but than relieving suffering. Nothing in medi- beneficent healer to
a few of the dangers that underlie in- cine should be more scrupulously avoid- do? Boning up on a
tractable pain of any origin (Dubner, ed than making matters worse; hence the knotty problem is always a good
1991; Liebeskind, 1991). dictum, First, do no harm. Nothing in beginning.
Why Pain? Because mismanagement of medicine better displays the tension be-
This special issue of RAMifications ex-
pain by the individual or the provider may tween these imperatives than the proper
amines the interface between the pharma-
lead to infections, loss of muscle mass treatment of pain.
cological treatment of pain and the field
and physical strength, progressive physi- Nor are the risks all borne by the patient. of substance-abuse medicine. Articles by
cal deterioration, and a slow spread of the In recent years, physicians have been experts in various disciplines will review
pain to other areas, eventually involving sued, disciplined, or prosecuted, not only the pathophysiology, psychology, and
the entire body or, at least, large parts of it. for overprescribing or otherwise misus- pharmacology of pain and substance
The social cost of the pain problem trans- ing, abusing, or diverting opioids, but abuse, the rational use of pain relievers,
lates into broken marriages, alcoholism, also for underprescribing, the latter and the distinctions between drug-seek-
and family violence. The economic cost doubtless occasioned by fear of the ing and relief-seeking behavior, as well
of chronic pain translates into absen- former. Excessive zeal for avoiding even as administrative, legal, and law-enforce-
teeism and job loss. The human cost of the appearance of impropriety leads to ment aspects of pain therapy.
chronic pain includes suffering, depres- the opposite error of permitting needless
sion, and suicide. There is a prevalent perception that the
suffering.
line between use and abuse of opioid
For all of these reasons and many more, Opioids and related drugs clearly relieve pain relievers is finer than it really is. In a
we owe it to ourselves and our patients to suffering when used appropriately. Just as perfect world, of course, there would be
understand pain management better. clearly, they pose threats of toxicity and no need for such drugs. In the next best
addiction when mishandled. The chal- world, all use of opioids would be legiti-
lenge is to treat pain effectively while mate, and the patients who really need
Karsten F. Konerding, MD
President
avoiding complications and staying out of them could reliably get all they need.
Deborah Love That sort of world should be our aim.
Executive Director
Lara Watson
Editorial Assistant
Duncan MacIvor, MD
Physician Editor

Christina Weber The Richmond Academy of Medicine Board of Trustees extends its
Layout
appreciation to our distinguished honorary editor for his help in
The Richmond Academy of Medicine
1200 East Clay Street this special publication:
Richmond, Virginia 23219
(804) 643-6631 Fax (804) 788-9987
Stephen P. Long, MD
Letters to the editor and editorial
contributions are encouraged, Commonwealth Pain Specialists
subject to editorial review.
Associate Clinical Professor of Anesthesiology
Nonmember subscriptions are Virginia Commonwealth University
available for $20/year.

Richmond Academy of Medicine

2
May 2002
R A M I F I C AT I O N S

You, Your Patient, and Purdue


Purdue has established the Partners Against Pain program to foster an alliance among

physicians, nurses, pharmacists, and hospice personnel. In addition, Purdue and


Partners Against Pain strive to understand the challenges of patients and their
caregivers in the quest for relief from pain.
The Partners Against Pain program is designed to reach both caregivers and patients to
help ensure that every patient receives adequate pain relief.

Visit our website to learn more about pain


management and prevention:

1999, 2001, Purdue Pharma L.P., One Stamford Forum, Stamford, CT 06901-3431 C6147B&W 4/04

3
May 2002
R A M I F I C AT I O N S

PainPolitics and Public Perception:


Virginias Experience
Stephen P. Long, MD
Commonwealth Pain Specialists
Associate Clinical Professor of Anesthesiology
Virginia Commonwealth University

Pain is a more terrible lord of mankind tion has been paid to a historically neg- pain.6, 7 Other internationally known opin-
than even death itself. lected, poorly understood, and inade- ion leaders, such as Drs. Katherine
Albert Schweitzer quately treated area of patient care. Fol- Foley, Russell Portenoy, Howard Fields,
lowing the publication of these acute and William Fordyce, have lent promi-
The word pain invokes great fear, not
pain guidelines, the Agency published nent voices and viewpoints. Recently,
only for patients, but also for providers.
similar guidelines concentrating on the professional societies, such as the Ameri-
We know intellectually that pain exists;
management of cancer pain.2 can Pain Society and the American Acad-
yet we often dont know how to relieve it,
emy of Pain Medicine, have issued con-
or we fear public/political scrutiny if we Despite the fact that, in both acute and
sensus statements publicizing the need
become outspoken advocates for its erad- cancer pain, the treatment remains pre-
for aggressive pain management. They
ication from our patients lives. From its dominantly the use of opioid analgesics,
continue to serve as professional re-
inception over 20 years ago, the Ameri- the need for other analgesic adjuvants,
sources for elected officials and third-
can Pain Society has sought to focus the both pharmacologic and nonpharmaco-
party payors.8
enthusiasm and energy of health care logic, is also clear. The universal usage
providers toward appropriate pain man- of opioid analgesics in these conditions Unfortunately, at the legislative level,
agement, attempting to reverse years of has led to much discussion and research little has been accomplished in terms of
neglect and disregard of this vital public on their role in treating chronic non- pain and its global social and financial
health field. Over the last decade, Vir- cancer (sometimes referred to as non- implications. Most legislation has dealt
ginia has been a national leader in the de- malignant) pain. Despite studies show- with broad issues in a general sense. In
bate over pain management, providing ing their efficacy in chronic pain, legal, particular, the issue of opioid use in
direction and stimulating dialogue, and educational, social and historical barriers chronic noncancer pain has been ad-
formulating policy. often preclude patients and prescribers dressed repeatedly, but superficially, by
alike from using them.3 often-conflicting state and federal laws.
The last several years have seen tremen-
For example, prescribing heroin to termi-
dous interest on the part of patients, Historically, sparse legislative and public
nal patients was legal in Virginia until a
third-party payors, state and federal gov- policy attention has been devoted to pain
recent prohibitive federal law superseded
ernments, licensing organizations, and medicine, especially in the area of opioid
Virginias law. Laws also may fail to re-
physicians of all specialties in the area of regulation. In the late 1980s, pioneers in
flect the current opinion held by state
pain medicine and the management of opioid regulation, e.g. Stratton Hill,
medical societies or regulatory boards.
patients with acute, chronic noncancer- brought public and legislative awareness
By the mid-1990s, several states had ap-
and cancer-related pain. The emergence to the concerns surrounding the proper
pointed official pain commissions.9
of pain medicine as a specialty in its prescribing authority of opioids. Massa-
own right has brought to light the impor- chusetts and Texas were leaders in ad- Virginia was one of the first states to seri-
4, 5
tance of education and proper treatment dressing these important concerns. ously address the topics of acute, chronic,
of pain in all spheres. David Joranson of the University of and cancer pain through a comprehensive
Wisconsin has devoted numerous articles task force-like group. The 1994 Session
Since the initial publication in 1992 of
and much professional time to examining of the General Assembly enacted Senate
the Agency for Health Care Policy and
historical and legal aspects of opioid pre- Joint Resolution 72 and, in cooperation
Researchs clinical practice guideline for
scriptions for all types of intractable with the Governors office, appointed a
acute pain management,1 enormous atten-

4
May 2002
R A M I F I C AT I O N S

joint subcommittee to study Virginias understudied area of chronic pain. Public was far from complete and decided to
laws and policies related to acute and support for the work of the committee re- continue its work into 1998-99 in order to
cancer pain management. The group was mained enthusiastic and strong. In partic- resolve the legislation enacted in the pre-
chaired by then-Senator Jane Woods, and ular, the group studied the economic im- ceding four years.
consisted of several House and Senate pact of chronic pain, as well as third-party
The joint subcommittees accomplish-
members, public lay members, and three coverage for pain management. Comple-
ments are too numerous to list, but the
physician members (Drs. Stephen P. mentary care techniques also were studied
culmination of the Virginia effort has
Long, John C. Rowlingson, and Vincent in detail, including the availability and ac-
been the unprecedented adoption by the
Speckhart). The eleven-member subcom- ceptance of such techniques in the health
Medical Society of Virginia, the state leg-
mittee engaged in a lengthy study of all care community. The group received testi-
islature, and the Board of Medicine, of
facets of acute and cancer painbasic mony describing that process from the
the Medical Society of Virginias Guide-
and clinical research; medical education; chairman of the Workers Compensation
lines for the Use of Opioids in the Man-
patient access to care; payor willingness Commission.
agement of Chronic, Non-cancer Pain.
to cover pain-related expenses; and the
One of the most significant areas ad- This document, the first of its kind in the
legal, regulatory, and social barriers to
dressed in the 1996 session was the some- United States, allows physicians to more
adequate care.
times-controversial use of opioids in the comfortably, safely, and intelligently pre-
The first year of study was spent gathering management of chronic noncancer pain. scribe proper analgesics to sufferers of
information. The group heard testimony The group heard much testimony about chronic pain with less fear of regulatory
from nationally known clinicians and re- its efficacy and limitations. In addition, repercussion.10, 11
searchers, conducted site visits to pain fa- the Virginia Board of Medicine, led by its
The document reflects the view of Vir-
cilities, and received input from insurance executive director, Dr. Warren Koontz,
ginianspatients, physicians, legislators,
carriers (e.g., Medicaid, Medicare, private took a preemptive stance in educating
administratorsthat physicians have an
third-party payors, managed care Board members on the proper use of opi-
obligation to treat patients with in-
providers) and state regulatory agencies oids to treat patients with chronic non-
tractable pain, reaffirming that opioids
(e.g., State Police, Boards of Medicine and cancer pain. The Board became a national
may be appropriately and safely pre-
Pharmacy). By the end of the first year of leader with the Federation of State Med-
scribed for many pain conditions pur-
study, it was apparent that further public ical Boards of the United States in provid-
suant to acceptable protocols and stan-
information sessions were required, and ing information about the Virginia effort
dards. While the guidelines are fairly
that Virginia should assume a national on both political and medical fronts. Con-
specific, the intent was to produce a fluid
leadership role in emphasizing the impor- currently, the Medical Society of Virginia,
document to serve, not as a definitive
tance of pain medicine. encouraged by the joint subcommittee
practice standard, but as a plastic, dynam-
and through resolution, appointed a spe-
In 1995, a legislatively mandated historic ic statement, providing the practitioner
cial committee to develop and implement
symposium on pain management was with a template for the logical and safe
comprehensive guidelines. Due to inade-
held in Richmond. Hundreds of partici- medical practice of prescribing opioids. It
quate education about pain, particularly
pants from around the country engaged in is anticipated that these guidelines will be
among physicians, the subcommittee en-
active dialogue with national thought updated to reflect changing and expanded
couraged the Medical Society of Virginia
leaders in the field of pain management. attitudes and experience.12
to develop these guidelines, realizing that
Keynote speakers led discussions con-
the problem of pain treatment originates Inspired by Virginias accomplishments,
cerning federal guidelines for managing
at the fundamental level of education of many national organizations sought to ad-
acute and cancer-related pain. Other
health professionals, particularly physi- dress the undertreatment of pain. Realiz-
health care providers from all sectors, as
cians. The subcommittee requested Vir- ing the dangers of underutilizing opioids
well as legislators and business leaders,
ginias medical schools to study and re- in chronic pain, the American Pain Soci-
participated in panel discussions, focus-
port on the educational process. Because ety and American Academy of Pain Med-
ing on areas requiring further attention.
of the many unanswered questions and icine issued a consensus statement in
By 1996, it was obvious that the subcom- problems associated with the treatment of 1997 entitled, The Use of Opioids for
mittee should be expanded to include the pain, the subcommittee felt that its work Continued on page 6

5
May 2002
R A M I F I C AT I O N S

the Treatment of Chronic Pain.13 This diversion of narcotics became rampant at use of opioids from the standpoint of ad-
statement eased the friction between ad- the turn of the century. State and federal diction and diversion. Dialogue on this
vocates for aggressive pain treatment and officials began an aggressive public attack quandary must persevere.
others, such as law-enforcement agen- on the problem in the worst areas of
The fact that undertreatment of pain per-
cies, medical boards, and providers abuseup and down the Appalachian
sists, despite nearly three decades of ef-
themselves, who viewed more skeptical- Trail, from Alabama to Maine. This in-
forts to educate providers, underscores
ly the prescribing of opioids for chronic tense scrutiny has once again put Virginia
the tensions outlined above. Pain has
pain. A dialogue began that resulted in in the national limelight of pain medicine.
been identified as an important barrier to
policy changes.
Because a significant amount of drug traf- the good of health of not only the patient
In May 1998, the Federation of State ficking and diversion first occurred in but also the entire health care system. To
Medical Boards adopted guidelines en- Southwest Virginia, in May 2001, then- emphasize the scope of this problem and
dorsing the appropriate use of opioid Attorney General Mark L. Earley con- the urgency to fix the situation of un-
analgesics, to clarify the Boards posi- vened a Prescription Drug Abuse Task dertreating pain, the Joint Commission on
tion on pain control, specifically as relat- Force to study the paradox of balancing Accreditation of Healthcare Organiza-
ed to the use of controlled substances, to aggressive treatment against the problems tions, in 2000, mandated that all accredit-
alleviate physician uncertainty and to en- of patient, pharmacy, and physician diver- ed facilities devise a plan to educate, as-
14
courage better pain management. Many sion of opioids. While the impact of the sess, and treat patients suffering from
states have published their own opioid recommendations of the task force are yet pain.16
policy statements. unclear, national organizations are closely
Through the historical efforts of the joint
following Virginias efforts to maintain
Unfortunately, by the late 1990s, persis- subcommittee, the Medical Society of
this delicate balance. In fact, the Drug
tent hesitancy on the part of many physi- Virginia Pain Committee, the Board of
Enforcement Agency continues to advo-
cians to treat pain aggressively brought a Medicine, the state legislature, the offices
cate for the appropriate treatment of
state level issue to national attention. of the Governor and Attorney General of
chronic pain patients and encourages
Oregons physician-assisted suicide law Virginia, and of groups such as the Rich-
agencies to eliminate any gray areas that
caused the American Medical Associa- mond Academy of Medicine, Virginia
tend to enhance the illegal trade of opioids.
tion, the American College of Physicians, continues to be a national leader and role
the Department of Veterans Affairs, and Congress recently has held hearings at- model, by supporting and recommending
others to launch aggressive campaigns to tempting to remedy the problem. Virginia educational and legislative activities to
ensure that physicians are sensitive to the continues to play a prominent role. The raise the level of awareness of the
needs of chronic and terminal pain pa- jury is still out concerning how patients provider community with respect to pain
tients. The Pain Relief Promotion Act of can be best protected, physicians less and its management. It is widely recog-
1999 was designed to override the assist- threatened, and pain preemptively and ag- nized that pain, not unlike all of medical
ed suicide laws of Oregon. While this bill gressively eliminated with minimal illicit care, cannot be legislated. Neither can
was not enacted, it had the effect of fo- involvement. medical care operate independently of
cusing attention on those who suffer in- laws necessary to ensure the protection of
Yet, it is omnipresent in 2002 that pain
tolerably. Congress and other administra- the public. Virginia realizes that it can
continues to be undertreated, underdiag-
tive agencies, particularly the Drug make pain management more accessible
nosed, and misunderstood. Pain remains
Enforcement Agency, became engaged in to its citizens, by taking steps to ensure
the primary reason that patients seek
the debate as the decade of pain manage- that (1) patients are treated by practition-
medical treatment. The majority of
ment began in 1999. ers who endorse contemporary methods
medical students, however, have limited
of pain therapy, (2) third-party payors in-
Part of the reason for this national offen- training in the appropriate treatment for
clude payment for pain specialty treat-
sive was the perceived and actual abuse these patients.15 Similarly, most law-
ment and interventions, and (3) dialogue
of Schedule II opioids throughout the enforcement officials and legislators
and education continue to occur between
eastern United States. While the national have equally limited understanding of
health care providers and law-enforce-
health care pendulum was swinging to- the tremendous financial, physical, and
ment officials.
ward the more liberal use of opioids for psychological impact which stems from
chronic pain, illegal trafficking and the undertreatment of pain, and view the

6
May 2002
R A M I F I C AT I O N S

The cost of pain is astronomicalin 3R.K. Portenoy, Opioid therapy for chronic 11S.P. Long, MSV house of delegates passes
human, economic, psychosocial, and nonmalignant pain: a review of the critical opioid guidelines, 125 VA. MED. Q. 8-11
issues, 11 J. PAIN & SYMPTOM MGMT. (1998).
legal terms. All patients should expect to 203-17 (1996). 12W. Koontz, Guidelines for the use of opi-
receive appropriate pain therapy. 4Medical Practice Act of Texas art. 4495(c) oids in the management of chronic, non-
Providers must not be intimidated by cur- (1989). cancer pain, 125 VA. MED. Q., supra,
rent legal public opinion and misconcep- 5Mass. Bd. Reg. in Med., Prescribing prac- at 11.
tion when it comes to appropriate thera- tices policy and guidelines adopted, 13See source cited supra note 8.
py. Virginia will continue to address the 4 News 1-2 (1989). 14FEDN ST. MED. BDS., MODEL GUIDELINES
6D.E. Joranson, Intractable pain treatment
problem of pain at all levels, and looks FOR THE USE OF CONTROLLED SUBSTANCES
laws and regulations, 5:2 A.P.S. BULL. FOR THE TREATMENT OF PAIN 1 pmbl.
forward to sharing its experience, both 1-17 (Mar./Apr. 1995). (May 2, 1998), available at
perceptively and politically, with those 7D.E. Joranson, State medical board guide- http://www.medsch.wisc.edu/painpolicy/
willing to recognize the problem.17 domestic/model.htm.
lines for treatment of intractable pain,
5:3 A.P.S. BULL. 1-5 (May/June 1995). 15N.G. Hoffmann et al., 19 J. ADDICTION
References DIS. 1-12 (2000).
8Am. Acad. Pain Med., The Use of Opioids
1U.S. DEPT HEALTH & HUMAN SERV., 16P.H. Berry, J.L. Dahl, Making pain assess-
for the Treatment of Chronic Pain 1-4
AGENCY FOR HEALTH CARE POLY & RES. (Spring Spec. Supp. 1997) (consensus ment and management a healthcare priority
PUB. NO. 92-0032, CLINICAL PRACTICE statement). through the new JCAHO pain standards,
GUIDELINE ACUTE PAIN MANAGEMENT: OPER- 8:2 J. PHARM. CARE IN PAIN & SYMPTOM
9S.P. Long, The work of Virginias joint sub-
ATIVE OR MEDICAL PROCEDURES AND TRAUMA
CONTROL 5-20 (2000).
(Rockville, Md., Feb. 1992). committee studying laws and policies relat-
ed to acute and cancer pain, 6:3 A.P.S. 17S.P. Long, Painthe public, the politics,
2U.S. DEPT HEALTH & HUMAN SERV., AGENCY
BULL. 18 (May/June 1996). the perception: the Virginia experience, AM.
FOR HEALTH CARE POLY & RES. NO. 94-
10S.P Long, Preliminary report of ad hoc PAIN SOC. BULL. (May-Aug. 1998).
0592, CLINICAL PRACTICE GUIDELINE NO. 9:
MANAGEMENT OF CANCER PAIN (Rockville, committee: pain management, 124 VA.
Md., Mar. 1994). MED. Q. 222-23 (1997).
Trigon Blue Cross Blue Shield is the trade name of Trigon Insurance Company An Independent Licensee of the Blue Cross and Blue Shield Association 2002

The marks of a leader.


7
May 2002
R A M I F I C AT I O N S

Modern Treatment of Chronic Pain


John Michael Barsanti, MD
Commonwealth Pain Specialists, LLC

In order to understand and appreciate the The modern tenets of pain management cal, components of pain. The physiologi-
rapid advances in pain management, a have challenged successfully, and hence cal aspects include understanding the cel-
historical perspective is needed. In 1992, changed, the practice of medicine. We lular mechanisms of pain, the pain path-
the American College of Graduate Med- now understand that there are many opi- ways in the spinal cord and brain, and
ical Education granted the specialty of oids from which to choose, and that the identifying the pain generator. To un-
anesthesiology a subspecialty board in use of alternative agents has allowed re- derstand the generator is the key to treat-
pain management. This coincided with duced dosing of those opioids. Agents ment. An analogy is to think of a
the birth and subsequent proliferation of such as injectable NSAIDs, anticonvul- chemotherapeutic agent, given systemi-
pain management physicians and pain sants, antidepressants, and sodium chan- cally, which nonspecifically kills cells.
clinics. Prior to this, pain physicians and nel blockers allow physicians to match We are learning of new smart drugs
clinics were few and predominately locat- the drug to the type of pain, thereby which can specifically target the offend-
ed in the larger teaching hospitals. Most avoiding the use of opioids exclusively. ing tumor. In the same way, we have
physicians and nurses in the community In the opioid class, most physicians are treated pain of all kinds with oral and
believed that they could adequately treat moving away from meperidine because systemic opioids, whose site of action is
pain; yet they were using outdated tech- of its active metabolitenormeperi- in the brain, with the purpose of masking
niques that subsequently were proven to dinewhich causes seizures in high or dulling perception of the pain. Ideally,
be ineffective. This is not surprising con- serum concentrations. We are now more we want to target our treatment precisely
sidering the paucity of classroom time de- comfortable using morphine, hydromor- to the source of pain. This targeted ap-
voted to pain management in nursing and phone (Dilaudid), and even fentanyl. proach of pain management, and the use
medical schools. As physicians, we re- of more advanced techniques, in fact,
Our dosing schedule of opioids has
ceived most of our training in pain man- takes us even farther away from tradition-
changed from PRN to round the clock.
agement during our year of internship. al opioid therapy. Modern treatment of
If the opioids duration of action is four
We were taught that opioids should be pain relies on the use of precision injec-
hours, then our schedule is for every four
used on a chronic basis only for malig- tion techniques and neuromodulation of
hours and not six. We now use oral and
nant pain. In acute settings, opioids pain through implantable devices.
transdermal sustained release opioid
should be used sparingly, and on a PRN I. Precision Injection Techniques
products, which can achieve steady
basis, not a scheduled basis. The typical A. Spinal pain
serum levels and improve compliance.
route chosen was intramuscular; other Spinal anatomy can be divided conve-
These are the same factors that have pop-
less traumatic routes (such as PO or intra- niently into three compartments: the pos-
ularized once-daily medications in other
venous) were not considered. We were terior, neuraxial, and anterior compart-
categories.
taught to use only one or two opioids, ments. Each compartment can be a
meperidine or morphine, and our PRN Physicians aggressive treatment of pain,
source of chronic neck or low back pain,
schedule was based on tradition rather with the right drug at the right time, has
and the pain often originates from more
than sound pharmacologic principles. We been a major achievement. Pain medi-
than one compartment.1
accepted the fact that, should the patient cine, however, involves more than phar-
complain or request more pain medicine macologic interventions. At the core of The posterior compartment contains all
than prescribed, he or she was either the specialty is the understanding of the the structures innervated by the dorsal
weak or drug seeking. physiological, as well as the psychologi- ramus of the spinal nerve and posterior to
the plane of the transverse process. Pain

8
May 2002
R A M I F I C AT I O N S

from the posterior compartment origi- quency to lesion the medial branch of noninvasive procedure of IDET ( intradis-
nates from the paravertebral muscles the dorsal primary rami. cal electrothermal therapy), as well as
and/or the zygapophyseal joints, referred surgical fusion.
The neuraxial compartment contains all
to simply as facet joints. Treatment of
structures within the epidural space and B. Sympathetic pain
the muscular component of back pain in-
neural pathways. To localize neuraxial Many pain syndromes involve some com-
cludes physical therapy, muscle relax-
pain, the diagnostic goal is to selectively ponent of the sympathetic nervous sys-
ants, mobilization, manipulation and
place anesthetic and corticosteroid into a tem. Most notable is the diagnosis of Re-
trigger-point injections. The facet joints
limited area of the epidural space. The flex Sympathetic Dystrophy, now known
are a common source of low back pain,
traditional epidural technique is a as Complex Regional Pain Syndrome.
but are often ignored. There is little dis-
translaminar approach, directing the nee- The use of injection techniques to block
pute that the joints undergo significant
dle between the lamina, and injecting a the sympathetic nervous system is the
degeneration as aging occurs. In fact, de-
large volume of steroid and local anes- treatment of choice in pain disorders in-
generative changes were found in two-
thetic into the epidural space, hoping to volving the sympathetic nervous system.
thirds of patients under the age of 30 in
bathe the inflamed nerve root with
cadaveric specimens.2 Disc space nar- II. Neuromodulation
steroid. Unfortunately, the injectate may
rowing, secondary to degeneration, can A. Spinal cord stimulation
never reach the target. With fluoroscopi-
increase the normal weight-bearing load Spinal cord stimulation can be used for a
cally guided transforaminal injections,
on the facet joints from a normal 3-25% variety of painful disease states. The most
the pain physician can place a needle at
to as much as 47%.3 For many patients common use in the United States is for
the exact foramen of the injured nerve
with degenerative disc disease, the discs patients with low back, leg, or arm pain
root and precisely place the steroid where
clearly can be a source of pain. A land- following unsuccessful lumbar or cervical
needed. This technique aids in both the
mark study by Mooney and Robertson spine surgery. It is also effective for a va-
diagnosis and treatment of common
reported that injection of saline into the riety of neuropathic pain conditions, in-
radicular symptoms.
facet joints of normal volunteers pro- cluding complex regional pain syndrome
duced back and leg pain that subsequent- The anterior compartment contains the and diabetic neuropathy. The use of
ly could be relieved by local anesthetic vertebral bodies and the intervertebral spinal cord stimulation is evolving to
injection.4 The ability to diagnose facet discs. The intervertebral disc can be a treat other diseases, such as intractable
joint pain early can lead to treatment source of axial low back pain, referred to angina, chronic pelvic pain syndromes,
with corticosteroid injection into the as discogenic pain. Lumbar discogenic and even stress incontinence. Typically,
joint, with quick resolution of pain. On pain presents as axial pain with no radic- the use of spinal cord stimulation is re-
physical exam, pain with back extension ular component. There should be no evi- served for patients with severe, unre-
and lateral rotation, and localized par- dence of disc herniation on MRI. The use lieved pain, combined with more conser-
avertebral muscle tenderness, are com- of discography can help identify the disc vative treatments such as medications and
monly seen with facet pathology.5, 6 There as the source of low back pain.8 When injections.
should be no evidence of nerve root irri- contrast is injected into the disc, both the
B. Intrathecal drug delivery
tation, such as MRI findings of a herniat- chemical stimulus of the contrast, as well
In the most severe pain disease states, the
ed disc or physical exam findings consis- as the mechanical stimulus of the injec-
use of conventional therapies fails. In
tent with a radiculitis. Unfortunately, the tion, can elicit pain. At least one control
some patients, despite aggressive use of
diagnosis cannot be made by abnormal disc, one that has been injected with con-
oral and parenteral opioids and nonopi-
findings on MRI or CT scans.7 The gold trast without eliciting pain, should be in-
oids, adequate pain relief with an accept-
standard in the diagnosis of facet joint jected. Typically, a provocative disc in-
able side effect profile cannot be achieved.
pain involves joint injection. Should the jection indicates disruption of the annular
In these cases, the use of intrathecal ther-
pain relief with corticosteroid injections wall. The patient generally describes pain
apy is necessary. Opioids are the first
be short-lived, the pain physician then similar to his or her typical pain, and CT
choice for intrathecal use, and morphine
can target the nerves that innervate the scanning post injection can demonstrate
is the most common agent. In addition,
joint. Six to twenty-four months of pain tears in the annular wall. The treatment of
pain practitioners have found that other
relief can be achieved by using radiofre- discogenic pain includes the relatively
Continued on page 10

9
May 2002
R A M I F I C AT I O N S

agents can be added to the opioid to in- sary for chronic nonmalignant pain. In an 3K.H. Yang et al., Mechanism of facet-load
crease the analgesia and thereby reduce effort to reduce the need for chronic opi- transmission as a hypothesis for low back
pain, 9 SPINE 557 (1984).
the amount of opioid required. Marcaine, oids, we have developed precision injec-
4V. Mooney & J. Robertson, The facet syn-
a local anesthetic, is commonly adminis- tion techniques and neuromodulation
drome, 115 CLIN. ORTHOP. 149-56 (1976).
tered intrathecally with morphine to aug- techniques, such as spinal cord stimula-
5T. Helig et al., The lumbar facet syndrome.
ment the analgesic effect. Clonidine also tion. Modern treatment of pain manage- 13 SPINE 61-64 (1988).
has been shown to be effective intrathe- ment attempts to target the source of 6A. Schwarzer et al., Pain from the lumbar
cally, especially with neuropathic pain pain and eradicate it, using all the tech- zygapophysial joints.
syndromes. niques and pharmaceuticals in our 7S.W. Wiesel et al., A study of computer as-
armamentarium. sisted tomography: The Incidence of posi-
The advances in pain management over tive CAT scans in an asymptomatic group
the past 20 years have been astounding. References of patients, 9 SPINE 549-51 (1981).
Our knowledge of the physiological basis 1R. Derby et al., Precision Percutaneous 8J. Saal, Management of Chronic Discogenic
of pain and its pathways is expanding. We Blocking Procedures for Localizing Spinal Low Back Pain with a Thermal
Pain, 3 PAIN DIG. 175-88 (1993). Intradiscal Catheter, 25:3 SPINE 382-88
have specific medications, opioids and (2000).
2D. Tobias et al., Human facet cartilage:
nonopioids, to target both the specific type
Swelling and some physico-chemical char-
of pain as well as the pain generator. We acteristics as a function of age, 17 SPINE
now accept opioids as sometimes neces- 694-700 (1992).

Membership Application Richmond Academy of Medicine Inc.


1200 East Clay Street Richmond, VA 23219 Phone (804)643-6631 Fax (804)788-9987

Name: __________________________________________Social Security #: _______________________


Group name____________________________________Solo practice: Y___ N___
Office address: _______________________________________________________________________
________________________________________________________________________
Phone: ______________________ Fax: ____________________________
Specialty: ______________________________________ Office manager __________________________
E-mail adddress:_______________________________ Birth date: ____/____/____ Sex: M___ F___
Please indicate the member (if any) who contacted you regarding membership:______________________

PERSONAL
(The information below will not be available to the public.)
Home address :____________________________________ zip code: __________
phone:_____________
Name of spouse: _______________________
I agree to be governed by the Constitution and By-Laws of the Richmond Academy of Medicine.

__________________________________________
Signature of applicant

Please send completed application to: Richmond Academy of Medicine, 1200 East Clay Street, Richmond, VA 23219.

10
May 2002
R A M I F I C AT I O N S

crease in pain levels. The brains natural


Chronic Pain from a Chemical pain-killing system, the endorphin-
Dependence Perspective enkephalin system, seems to be negatively
affected by chronic narcotic use. Because
Peter R. Coleman, MD of homeostasis, the brain seems to shut off
Clinical Assistant Professor
Medical College of Virginia its own pain-relieving system, causing
Medical Director pain to actually increase whenever the
Outpatient Substance Abuse Services brain level of narcotics starts falling.
MCV Family Counseling Center
Approach to chronic pain patients
Because 10% of the population has a
Chemical dependence is now recognized time drug dependence sets in, continued chemical dependence problem, it seems
as a brain disorder that affects roughly drug use is largely outside of conscious obvious that some chronic pain patients
10% of the general population without control. will have a problem with narcotics. Their
regard to gender, socioeconomic status, management will be more difficult and
Chronic pain and withdrawal states
race, or profession. The most useful defi- their ability to control drugs and not suffer
Chronic pain is a condition that has many
nition of chemical dependence is that it negative consequences will be severely
causes. It frequently leads to continued
is a disorder characterized by continued impaired. My recommendations for the
narcotic use and then a state of physical
use of mood altering substances, in spite approach to chronic pain patients includes
dependence on opiate drugs. When narcot-
of negative consequences caused by them. the following:
ic use is stopped, a withdrawal syndrome
The cause seems to be a complex inter-
sets in. Narcotic withdrawal is character- 1. Obtain the best possible history, physi-
play of bio- and psychosocial factors, but
ized by the opposite of the acute narcotic cal and special test results. If necessary,
the most powerful factor seems to be ge-
effects. There is total body pain, diarrhea, refer to specialists in order to make the
netic. Approximately 50% of the children
dilated pupils, sweats, cold chills, and in- most accurate diagnosis of the true pain
of alcoholics or drug addicts will have a
tense dysphoriaa profound sense of total state. It is important to determine the
chemical dependence problem. There also
discomfort. Most addicts are unable to tol- exact nature, expected severity and du-
are animal studies that demonstrate strong
erate this on their own, and the success ration of the pain, and what drugs are
genetic factors affecting the severity of
rate for enduring the withdrawal period is usually needed for these patients.
withdrawal symptoms. In situations of
very low. There appear to be three phases
chronic pain, there is also some evidence 2. Determine if there is any past history of
to withdrawal. The first is acute withdraw-
that continued substance use may, over a substance use or abuse. What is the pat-
al, which lasts from five-to-ten days and
period of time, cause permanent brain tern of narcotic and other mood-alter-
has the most intense withdrawal symp-
changes leading to chemical dependency. ing substance use in this episode of dis-
toms. Following this, there appears to be
The parts of the brain affected by chemi- ease? Has there been past use, abuse, or
a subacute period of slow improvement,
cal dependence include the locus ceruleus negative consequences with narcotics,
lasting one-to-eight weeks and consisting
and areas generally referred to as the alcohol or other drugs? Has there been
of a powerful sense of fatigue, depression,
pleasure center. An understanding of the any past attendance at NA, AA, coun-
and insomnia. The third phase, or protract-
power of chemical dependency may be seling, or other drug treatment pro-
ed withdrawal, is mild, but persists with
gained from studies in which animals are grams? Is there a family history of sub-
low-level symptoms up to two years.
given free access to a drug, such as co- stance abuse? Usually, it is necessary to
Withdrawal severity is extremely variable.
caine. Almost all of these animals will die interview family members to obtain the
It appears that people who have a genetic
of starvation within a thirty-day period. most accurate information to these
tendency for chemical dependency have a
Thus, chemical dependence is a disorder questions.
much stronger attachment of narcotics to
of the primitive brain. The animals desire
the brain receptors and a much more in- 3. Decide on a course of action in treating
for the drug effect is so strong that it over-
tense withdrawal. this pain. If there is evidence of a past
powers basic biological drives, including
Unfortunately, in some instances, chronic or present chemical dependency prob-
the drive to eat and sustain life. While ini-
tial drug use may be a willful act, by the narcotic use seems to actually create an in- Continued on page 12

11
May 2002
R A M I F I C AT I O N S

lem, all attempts should be made to use call the police if there is evidence of legal implant a specially formulated preparation
nonnarcotic pain medicines. If nar- improprieties. of naltrexone, a narcotic antagonist. This
cotics or other mood-altering drugs are pellet is implanted subcutaneously and
Pain use and recovering patients
needed, strict monitoring will be neces- dissolves slowly, guaranteeing abstinence
Frequently, patients will tell their doctors
sary, and patients should use the least from all narcotics for six-to-ten weeks.
that they are in recovery from chemical de-
abusable drugs possible. If there is a
pendency. Sometimes, these patients need Can chronic implantable Naltrexone
history of moderate abuse, I usually try
narcotics for short-term treatment, includ- actually treat chronic pain?
to convince the patient and family to try
ing acute surgery, kidney stones, dental Naltrexone is a drug that blocks the endor-
nonnarcotic use as much as possible. I
procedures, etc. In my experience, when phin receptor system. It will precipitate
also try to make them understand and
patients are truthful about these needs and immediate and complete withdrawal if any
agree that, if they will withstand some
are willing to follow through with a couple opiates are left attached to the receptors,
current discomfort, their long-term pain
of simple suggestions, they have no prob- and it will then completely block any nar-
levels will decrease. If there is no evi-
lem getting readdicted to medicines. cotic effect, if narcotics are used. For over
dence of abuse, it is easy to use nar-
Specifically, I involve either a spouse or an three years, I have been performing accel-
cotics, but remember that uncontrolled
AA support person to monitor the frequen- erated detoxifications of patients who have
chronic narcotic use may lead to physi-
cy of medicine use, and I make sure the chronic pain and are physically dependent
cal dependence and addiction.
prescribing physician is aware of their past on their narcotics. Usually, these patients
Nonnarcotic treatment modalities problem. They should go home with as know they have had a chemical dependen-
Nonnarcotic modalities vary depending small a prescription as possible. I schedule cy problem and are aware that they are
on the cause of the pain. Chronic painful frequent visits or phone calls in the first now abusing the drugs and that the origi-
headaches may be treated acutely with week or two, merely to ask how many nar- nal pain is no longer particularly severe. I
triptans, nonsteroidals, Tylenol, self-injec- cotics they are using. We use maximum have been completing their detox in one-
tions of Toradol, Trilafon, and other drugs. doses of nonnarcotic analgesics and have to-three days and then implanting a form
Prophylactic medicines for headaches are the patients stop narcotics as soon as possi- of naltrexonea pellet that slowly dis-
well outlined in literature. Chronic back ble. In my experience, when people do solves subcutaneously and provides nal-
pain usually may be managed with inten- these simple things, there is hardly a prob- trexone therapy for six weeks. I have per-
sive physical therapy, epidural steroids, lem with anyone becoming readdicted to formed this procedure in over twenty
trigger-point injections, acupuncture, re- narcotics. patients who have had various forms of
laxation techniques, accupressure, etc. chronic pain, including back pain, mi-
New detox methods
graines, trigeminal neuralgia, and RSD. In
When the decision to use narcotics is The standard detox method for narcotics is
every case, patients have completely
made, these nontreatment modalities to slowly taper the narcotics over one-to-
stopped taking their narcotics and have re-
should be chosen to minimize the chance eight weeks. Unfortunately, this is fre-
ported that their pain is either dramatically
of abuse. Generally, the longer acting quently unsuccessful because it is still very
improved or is no worse. All of these pa-
drugs, while having a longer withdrawal painful, and patients often cannot tolerate
tients are delighted to be free of narcotics.
state whenever detoxification is needed, the discomfort, especially when they know
They have all reported an increased sense
are also less abusable. Therefore, that taking a few more narcotics will make
of energy and clarity of thought, and dra-
methadone is probably the best choice. them feel much better. I have now devel-
matic improvement in their depression and
Fentanyl patches may also be useful. Fre- oped an outpatient detox method that ac-
overall well-being. Almost always, their
quently, a combination of a long-acting celerates that process, decreases the dis-
pain is dramatically less than when they
narcotic, such as methadone, with small comfort, and has a 95% successful detox
were on large doses of narcotics. My sense
amounts of short-acting drugs, like Perco- rate. By treating patients with large doses
is that, once we implant the naltrexone, it
cet, may be helpful for breakthrough pain. of Buprenex and benzodiazepines, I can
removes their ability to use narcotics and,
If there is evidence of prescription abuse, I complete the detox in two-to-three days. It
thus, their brain stops craving. Their en-
usually shorten the interval between office is a similar process to the rapid detoxifica-
dorphin-enkephalin system is able to re-
visits. For serious abuse, such as forging tion under anesthesia (ultra-rapid detoxifi-
turn to normal.
prescriptions, visiting other doctors, etc., I cation) but is much safer and better tolerat-
ask them to seek help elsewhere or even ed by patients. At the end of the detox, I

12
May 2002
R A M I F I C AT I O N S

The Pharmacology of Opioids


Michael Weaver, MD Deborah Haller, PhD Corresponding Author:
Assistant Professor of Internal Professor of Psychiatry Martha J Wunsch, MD
Medicine and Psychiatry Department of Psychiatry Assistant Professor of Psychiatry
Department of Internal Medicine Medical College of Virginia Department of Psychiatry
Medical College of Virginia Virginia Commonwealth University Medical College of Virginia
Virginia Commonwealth University Virginia Commonwealth University

Opioidsnaturally occurring alkaloids opioids will evidence improvement of length of activity depending on the drug
derived from the poppy planthave long their overall quality of life; in contrast, itself, the specific formulation, and mode
been valued for their analgesic efficacy. the consequences of opioid abuse can be of administration. Heroin and morphine
They currently are available in natural devastating. To meet the diagnostic crite- have a rapid twenty-to-thirty minute
(morphine, codeine) and synthetic (hy- ria of addiction to opioids, or any psy- onset of action for both analgesia and eu-
dromorphone, fentanyl) formulations for choactive substance, an individual must phoria and a relatively short duration of
medicinal use and also are procured illic- also display compulsive use, continued action (three-to-four hours). When given
itly for abuse. The beneficial effects of use despite harm, impaired control over parenterally, morphine will provide anal-
opioids include analgesia, changes in the drug, and craving. gesia for two-to-four hours and, when
mood including euphoria, a general de- given in an oral controlled-release formula-
Opioids are active by binding to recep-
crease in anxiety, and a decreased re- tion, may be effective for twenty-four
tors on cell membranes in the central
sponse to adverse physical and psycho- hours. Methadone, a long-acting opioid,
nervous system, gut, and musculoskeletal
logical stimuli. Side effects of the opioids may require seventy-two hours of dosing
tissues. The primary receptors identified
include somnolence, mild cognitive diffi- to achieve a therapeutic level but will pro-
in humans are labeled mu and kappa.
culties, insomnia, muscle twitching, itch- vide analgesia for six-to-eight hours. Fen-
These are responsible for differing effects
ing, nausea, vomiting, and constipation. tanyl, short acting in the parenteral form, is
and are present in varying concentrations
Adverse effects on the autonomic nervous active for seventy-two hours when admin-
throughout human tissues. Opioids are
system include vasodilatation and subse- istered through a transdermal patch.
classified as full agonists or partial ago-
quent hypotension and depression of the
nists, reflecting efficacy and potency at The amount and frequency of dosing will
respiratory center. Respiratory depres-
these receptors. Morphine is a full mu ag- vary among patients. Dosage should be
sion, rare in the setting of a patient with
onist and, therefore, is very potent in in- titrated to an effective level of analgesia
severe acute pain, is more likely to occur
ducing analgesia and euphoria. Partial with minimum side effects. A patient
in the opioid-naive individual. Tolerance
agonists, such as buprenorphine, may who is still experiencing pain, but ap-
develops to both the beneficial and ad-
have less abuse potential due to a thresh- pears euphoric or sedated, may require a
verse effects of opioids, though at differ-
old effect. Antagonist opioids, such as change to a different opioid. Because of
ent rates. Acclimation to the euphoric
naloxone, precipitate withdrawal in phys- variability in doses among opioids, clini-
effects occurs before sedation and analge-
ically dependent individuals through re- cians must be aware of equianalgesic
sia, while tolerance to respiratory depres-
ceptor blockade mechanisms. Typically, dosages. If a patient requires a different
sion occurs much later. Physical depend-
they are not used for pain management opioid or route of administration, the cli-
ence, manifested by the development of
but, rather, for reversal of respiratory de- nician may calculate an equianalgesic
withdrawal symptoms upon abrupt cessa-
pression due to opioid overdose. dose and schedule based on known
tion of opioids, occurs after seven-to-ten
bioavailability. Opioids produce incom-
days of administration of the drug. A The bioavailability of opioids varies
plete cross-tolerance due to subreceptor
patient who is physically dependent on across preparations. While all are metab-
variability and individual patient differ-
opioids is not addicted. Patients with olized primarily in the liver and excreted
ences; therefore, when changing to a dif-
pain who are adequately treated with by the kidneys, they differ in onset and
Continued on page 14

13
May 2002
R A M I F I C AT I O N S

ferent opioid, it is best to start with two- in improved quality of life. Another con- When shifting to a different opioid
thirds of the calculated equianalgesic cern is co-morbid addiction. Although pa- drug, remember that there is not 100%
dosage. The patient may then be titrated tients with both pain and addiction may cross-tolerance between different opi-
up to a therapeutic dosage if necessary. benefit from opioid therapy, they require oids. Use equianalgesic conversions
The Table lists several short- and long- thorough evaluation and careful monitor- and reduce the dose by 33%.
acting opioids, their onset and duration of ing by an addiction specialist with knowl-
Do not administer an antagonist or ag-
action across formulations, and suggested edge of pain if they are to have good out-
onist/antagonist to a patient main-
dosage ranges. In addition, it lists the comes. In summary, an understanding of
tained on opioids, as it will abruptly
equianalgesic dosages across different the pharmacology of opioids, coupled
induce withdrawal.
drugs and routes of administration. with sound clinical principles, can result
in improved care for the patient with pain. Bibliography
When opioids are used to treat addiction,
the dosing differs. Patients who are being Clinical Bullets A.W. Graham, T.K. Schultz eds. Princi-
treated for pain will require multiple ples of Addiction Medicine. 2d ed. Chevy
The effective analgesic dose should be
dosages throughout the day to achieve Chase, Md: American Society of Addic-
determined by patient response and
therapeutic analgesic effects. Methadone tion Medicine, Inc., 1998.
side-effect profile, as there are no de-
must be given every six-to-eight hours to
fined maximum dosages (titrate the J.G. Hardman et al. eds. Goodman &
achieve analgesia. In contrast, in the
dose based on effect, not number of Gilmans the pharmacological basis of
treatment of opioid addiction, methadone
milligrams). therapeutics. 10th ed. New York: Mc-
is administered every twenty-four hours,
Graw-Hill Medical Publishing Division,
and patients achieve a suppression of The development of tolerance to opi-
2001.
craving without euphoria. Receptor activ- oids necessitates gradual tapering
ity at the cellular level is responsible for when they are discontinued.
this variation in efficacy. A patient main-
tained on methadone for addiction who
develops pain will not have adequate pain
control if provided only with a once-daily
dose of this opioid.
Pain patients using opioids may benefit
from the addition of adjuvant medica-
tions. Salicylic acid and acetaminophen
often are formulated with opioids for this
purpose. In addition, patients may benefit
from the addition of a COX-2 inhibitor.
For patients with neuropathic pain, mem-
brane stabilizers, such as gabapentin, car-
bamazepine, and valproate, may help
lower pain levels. Tricyclic antidepres-
sants and other antidepressants may also
improve outcomes.
Not all patients are candidates for opioid
maintenance therapy. Decreases in pain
severity in the absence of any improve-
ment in functioning are considered prob-
lematic, as relief from pain should result

14
May 2002
R A M I F I C AT I O N S

The Diagnosis and Treatment of Headache


Alan R. Towne, MD
Professor and Chair of Neurology
MCV Hospitals and Physicians of the
Virginia Commonwealth University

I. Epidemiology and Impact of Headache pain is usually unilateral, of a pul- B. New Headache Evaluation
Headache is a common and sometimes sating quality, moderate or severe i. Obtain a complete headache histo-
disabling condition which accounts for intensity, aggravated by exercise. It ry, including age of onset, frequen-
approximately 5%-10% of outpatient vis- usually is associated with nausea cy, precipitating factors, location,
its to primary care physicians and 20% of and/or vomiting, photophobia and severity, duration of pain, presence
outpatient visits to neurologists. Migraine sonophobia. It should be remem- of an aura, headache pattern, and
headaches affect 18% of women and 6% bered that not all the characteristics medications used during the pain.
of men in the United States. It begins be- need to be present to classify the Other things that should be consid-
fore the age of twenty in half of sufferers, headache as migraine. ered in the history include other
with a prevalence that is highest in ages chronic illnesses or conditions, al-
ii. Migraine without aura: This type
twenty-five to fifty-five. lergies or sensitivities to medica-
of migraine has all the characteris-
tions, sleep habits, family history
II. Establishing the Diagnosis tics of migraine with aura except
of headaches, and recent life
Headaches are classified as either pri- that the aura is not present.
stressors.
mary or secondary headaches. Primary iii. Tension-type headache: This type
headaches are benign, often recurrent and ii. Physical examination to evaluate
of headache is characterized by a
are not secondary with any known under- any potential red flags, such as
pressing/tightening (nonpulsating)
lying pathology. Secondary headaches confusion, weakness, sensory loss,
quality, which is of mild or moder-
are caused by an underlying medical con- pupillary asymmetry, papilledema,
ate intensity, usually bilateral, and
dition, such as infectious or intracranial Babinski response, signs of
generally not aggravated by exer-
pathology. meningeal irritation, progressive
cise. Nausea or vomiting is not
visual or neurological changes, and
A. Primary Headaches: The most com- present. Photophobia or sonopho-
other evidence to suggest an under-
mon primary headaches are mi- bia might be present but not
lying neurological disorder.
graine,with and without aura, cluster simultaneously.
headache, and tension-type headache. iii. Perform diagnostic testing, if ap-
iv. Cluster headache: This type of
propriate, including blood tests
i. Migraine with aura: This type of headache generally is seen in
such as CBC, ESR or C-reactive
migraine is associated with an aura males and occurs in clusters lasting
protein. Lumbar puncture if
that generally develops gradually 15-180 minutes, if untreated. It is a
pseudotumor cerebri or infectious
over greater than four minutes, severe unilateral, orbital or super-
etiologies are entertained. It is not
characterized usually by visual, orbital pain that is severe. It may
necessary to perform neuroimaging
sensory or motor symptoms. The be associated with conjunctival in-
procedures in all patients who
headache usually follows the aura jection, lacrimation, nasal conges-
present with headache. The pres-
but may also begin before, or si- tion, rhinorrhea, facial sweating,
ence of red flags, however,
multaneously with, the aura. The myosis, ptosis and eyelid edema.
may make the use of CT or MRI
indicated.

16
May 2002
R A M I F I C AT I O N S

iv. Determine working diagnosis and vii.Other nonpharmacological ap- c) Frequent headaches (> 2 attacks
differential diagnosis. proaches include physical therapy, per week).
ultrasound, transcutaneous electri-
v. Establish treatment goals and d) Uncommon migraine conditions,
cal nerve stimulation, and thera-
expectations with patient. such as basilar migraine.
peutic massage. Some of these
vi. Review diagnosis, treatment techniques are still in their experi- e) Patient preference.
plan, and specific instructions mental stages and have not been IV. Conclusions
with patient. shown to be effective in controlled
trials. Headache is a common condition that, in
vii. Arrange follow-up care.
some patients, is disabling and accounts
III. Managing the Patient with Headache B. Pharmacological Approaches for frequent use of medical resources.
i. Abortive therapies. (summarized in The burden of migraine is substantial and
A. Nonpharmacological Approaches
Table 1). The goal of abortive ther- greatest for the most severely affected
i. Counseling and psychotherapy in apy is to rapidly resolve the mi- sufferers. Despite improvements in diag-
patients with co-morbidity, such as graine attack, hopefully without re- nosis and effective treatments, migraine
depression and anxiety. Psychiatric currence and to restore the patients remains underdiagnosed and undertreat-
consultation may be indicated as ability to function. Daily or near ed. There are, however, many effective
an adjunct to medication. daily use of abortive medications treatments available.
ii. Relaxation techniques and bio- may not be recommended because Table 1, summarizing selective abortive
feedback. Biofeedback may be of possible liver, kidney or GI toxi- therapies, appears on page 18.
used either alone or in conjunction city. Frequent use of some abortive
therapies may lead to habituation Table 2, summarizing selected prophylac-
with other approaches, such as
or the development of rebound tic therapies appears on pages 19 and 20.
medication.
headache. Some headache experts,
iii. Exercise will help promote proper
however, use NSAIDs as both
sleep patterns and also may reduce
abortive and prophylactic agents.
headache frequency and intensity.
Acute migraine medications in-
iv. Neural blockade. Nerve blocks, clude NSAIDs, combination anal-
such as greater occipital nerve gesics, opioids, antiemetics, and
blocks, may be helpful in certain corticosteroids. Specific agents in-
patients. Studies have demonstrat- clude ergotamine and the triptans.
ed that the injection of botulinium The triptans are more effective in
toxin may reduce the intensity and migraine and cluster headaches
severity of headaches. than in tension-type headaches.
v. Acupuncture: Some studies have ii. Preventive therapies (summarized
demonstrated that patients may in Table 2). Prophylactic treatment
obtain benefits with this technique should be considered when:
when performed by properly
a) The migraine significantly inter-
trained practitioners.
feres with the patients daily rou-
vi. Limit factors that may influence tine despite acute agents.
the onset or severity of headaches,
b) The acute medications are con-
such as fasting/missing meals, hor-
traindicated, ineffective, or have
mones, excessive noises, and di-
intolerable adverse effects.
etary factors (ripened cheeses,
sausage, citrus foods, alcoholic
beverages, etc.).

17
May 2002
R A M I F I C AT I O N S
TABLE 1 SELECTED ABORTIVE THERAPIES
Medication
COMBINATION THERAPIES
aspirin, 250 mg.; acetaminophen, 250 mg.; 2 tablets stat (p.o.). May cause rebound
caffeine, 65 mg.
Isometheptene mucate, 65 mg.; 2 capsules stat, then 1 q 1 hour to a max of 5 capsules/24 hour
dichloralphenazone, 100 mg.; acetaminophen, 325 OR 2 capsules stat, then repeat 2 capsules in 1 hour. Generic
mg (p.o.)* versions may not be equivalent to Midrin
Butabital 1 tab q 4-6 hours p.r.n. May cause rebound headache
Hydrocodone/acetaminophen or 1 tab q 4-6 hours p.r.n. May cause rebound headache
oxycodone/acetominophen
NSAIDS
NSAIDS may be combined with migraine specific therapies
without vasoconstriction occurring
Acetylsalicylic acid 650 mg. stat (2 regular strength tablets) (p.o.)
Diclofenac 50-100 mg. (p.o.)
Flurbiprofen 100 mg. stat; repeat in 1 hour p.r.n.
Ibuprofen 1200 mg. stat; 600 mg. every hour p.r.n.; max 2400 max/day
Ketoprofen 100 mg. stat; 50 mg. p.r.n. in 1 hour
Ketorolac 60 mg. (IM); limit use to one dose
Meclofenamate 200 mg. stat; repeat x 1 after one hour (p.o.)
Naproxen Sodium 825 mg. stat; repeat 275 mg. in 1 hour p.r.n.
Rofecoxib 25 mg. stat; repeat in 4 hours p.r.n; max 50 mg./day
ACUTE ABORTIVES
**Ergotamine tartrate, 2 mg. sublingual One tablet under tongue at earliest sign of headache; may repeat
every 30 minutes to a max of 3 tablets/24 hours or 5
tablets/week
**Ergotamine tartrate, 1 mg., and caffeine, 100 Two tablets at earliest sign of headache; may repeat one tablet
mg. (p.o.) every 30 minutes, up to 6 tablets/24 hours, 10 tablets/week
**Ergotamine tartrate, 2 mg., and caffeine, 100 Begin with one-quarter to one-half suppository; may repeat dose
mg. (per rectum) in 60 minutes; max 4 mg./24 hours, and 10 mg./week with four-
day hiatus between treatment days
*Sumatriptan succinate, 6 mg. subQ injection 1 injection at onset of headache phase of attack; max
subcutaneous dosage 12 mg. (2 injections)/24 hours, at least 1
hour apart.
Sumatriptan intranasal 20 mg. intranasal stat; repeat in 2 hours p.r.n.; max 40 mg./day
*Sumatriptan succinate, 25 mg., 50 mg., and 100 25-100 mg. at onset of headache phase; greater response rates
mg. available (p.o.) occur with 50 mg. and 100 mg., and 100 mg. orally; if no
satisfactory response within 2 hours, repeat dose every 2 hours;
for recurrence, an additional p.o. dose may be repeated; oral
response rates somewhat lower than subcutaneous or intranasal
routes; max 300 mg./day
*Zolmitriptan, 2.5 mg. and 5 mg. available (p.o.) 2.5-5 mg. at onset of headache phase of attack; repeat dose every
2 hours p.r.n.; max 10 mg./day
ANTIEMETICS
Chlorpromazine, 50 mg. rectal suppository Give one time
Metoclopramide, 10 mg. (p.o.) or 10 mg. slow IV Give by itself or 10 mg. may be given (p.o.) 30 minutes prior to
push NSAIDS or ergots
Prochlorperazine, 10 mg. slow IV push or 25 mg. Give one time; IV administration more effective than rectal
rectal suppository suppository at these doses
OTHER
Oxygen 7-10 l/minute for approximately 10 minutes by face mask

*Triptans are contraindicated in presence of uncontrolled hypertension, history of MI, ischemic or structural
heart disease, cerebrovascular disease, peripheral vascular disease, hepatic or renal impairment, and basilar or
hemiplegic migraine. Triptans should not be used within 24 hours of treatment with ergot-type drugs and other
triptan drugs.
**Ergots are contraindicated in the presence of uncontrolled hypertension, history of MI, ischemic or structural
heart disease, cerebrovascular disease, peripheral vascular disease, hepatic or renal dysfunction, sepsis or
pregnancy, and basilar or hemiplegic migraine. Ergots should not be used within 24 hours of treatment with
triptans and other ergot-type drugs.

18
May 2002
R A M I F I C AT I O N S
TABLE 2SELECTED PROPHYLACTIC THERAPIES
Medication Doses/Route Clarification

Beta blockers with partial agonist activity have not May cause fatigue, depression, bradycardia, and decreased sexual ability.
been shown to have efficacy in migraine Avoid use in asthma, COPD, and CHF, heart block greater than first degree.
prophylaxis Check for numerous drug interactions
Atenolol 25-100 mg./day (p.o.)
Metoprolol 100-200 mg./day (p.o.)
Nadolol 20-120 mg./day (p.o.)
Propranolol 60-160 mg./day (p.o.)
Timolol 10-20 mg./day (p.o.)
CALCIUM ENTRY BLOCKERS May cause hypotension, constipation, nausea, flushing, light-headedness,
edema
Diltiazem 96-360 mg./day (p.o.)
Nimodipine 30 mg. t.i.d.
Nicardipine 20-30 mg. b.i.d.-t.i.d. (p.o.)
Verapamil 120-480 mg./day (p.o.)
NSAIDS
Acetylsalicylic acid One 325 mg. tablet every other day
Celecoxib 100-200 mg. b.i.d.
Fenoprofen 600 mg. t.i.d. (p.o.)
Flurbiprofen 100 mg. b.i.d. or t.i.d. (p.o.)
Ketoprofen 75 mg. t.i.d. (p.o.)
Nabumetone 1000 mg. (QD (p.o.))
Naproxen 250-500 mg. b.i.d. (p.o.)
Rofecoxib 12.5-25 mg. once daily
TRICYCLIC ANTIDEPRESSANTS Some drugs in this category have not been studied in controlled clinical
headache trials, although clinical experience shows good effect in selected
Nonsedating: patients. Cost and side effect potential should be considered
Desipramine 25-150 mg./day (p.o.)*
Protriptyline 5-30 mg./day (p.o.)*
May be given in divided doses, with the largest portion of daily dose given
Sedating: at bedtime
Amitriptyline 10-150 mg./day (p.o.)*
Doxepin 10-150 mg./day (p.o.)*
Imipramine 10-150 mg./day (p.o.)*
Nortriptyline 10-150 mg./day (p.o.)*
SELECTIVE SEROTONIN REUPTAKE Side effects may include nausea, diarrhea, insomnia, agitation, sexual
INHIBITORS dysfunction
Fluoxetine 10-80 mg./day (p.o.). Fluoxetine has not been shown to be effective in
migraine prophylaxis in a published clinical study*
Fluvoxamine 50-100 mg./day (p.o.)
Paroxetine 20-60 mg./day (p.o.)
Sertraline 50-200 mg./day (p.o.)
OTHER ANTIDEPRESSANTS
Bupropion 200-300 mg./day. Side effects may include CNS agitation and seizures*
(p.o.)
Nefazadone 200-600 mg./day (p.o.). Side effects may include nausea, constipation,
dizziness, dry mouth, fatigue, insomnia, asthenia and agitation*
Trazodone 50-300 mg./day. Use in males may result in priapism
Venlafaxine 75-225 mg./day (p.o.). Side effects may include nausea, constipation,
dizziness, dry mouth, fatigue, insomnia, asthenia, seizures, and agitation*

19
May 2002
R A M I F I C AT I O N S
TABLE 2 (CONT.)

ANTICONVULSANTS
Divalproex sodium 250-1000 mg./day (p.o.). Avoid in patients with liver disease; liver function
tests should be done prior to and during therapy if clinical symptoms
warrant; weight gain and alopecia possible; may cause pancreatitis and
polycystic ovaries; use with caution in women of child-bearing potential
Gabapentin 300-900 mg./day
Tiagabine 8-32 mg./day
Topiramate 25-150 mg./day. May cause renal stones
SPECIAL USE THERAPIES
Methylergonovine 0.2 mg b.i.d.-q.i.d. (p.o.); max before one month drug holiday is 4-6 months
Phenobarbital, 40 mg.; ergotamine tartrate, 0.6 1 tablet b.i.d. or 1 tablet q.h.s. (p.o.). May cause rash, fatigue, and dry
mg.; belladonna alkaloids (Bellafoline), 0.2 mg. mouth; may be utilized for menstrual migraine
Cyproheptadine Max 4-8 mg q.i.d. (p.o.); begin with 4-8 mg q.h.s., increase to maximum
dose p.r.n. Commonly used in children, often causes weight gain, sedation
and other anticholinergic effects
Methysergide 2 mg b.i.d.-q.i.d. (p.o.) max use before one month drug holiday 6 months.
Fibrotic complications, peripheral ischemia, hallucinations, and peptic
ulcer disease can occur
MAO Inhibitor: Phenelzine requires intensive 15-60 mg/day (p.o.). Significant food and drug interactions; ingestion of
patient education and cooperation large amounts of tyramine may result in hypertensive crisis, MI, or
cerebrovascular accident; allow 14 days between use of tricyclics or
other antidepressants; fluoxetine must be discontinued for at least 5 weeks
before initiating MAO Inhibitor therapy
Isocarboxazid 10-40 mg/day (p.o.). Significant food and drug interactions; ingestion of
large amounts of tyramine may result in hypertensive crisis, MI, or
cerebrovascular accident; allow 14 days between use of tricyclics or
other antidepressants; fluoxetine must be discontinued for at least 5 weeks
before initiating MAO inhibitor therapy
ALTERNATIVE THERAPIES
Feverfew Herbal therapies are difficult to predict for consistent efficacy due to lack of
standardization in the preparations; may interact with other herbal, over the
counter and prescription medications; should be used with the full
knowledge of the physician
Magnesium 400-600 mg/day
Riboflavin (Vitamin B2) 400 mg/day
*Must allow 14 days after discontinuing use of MAOs to begin use

Tables 1 and 2 from US Headache Consortium Guidelines

20
May 2002
Now we can enjoy our lives together,
without chronic pain getting in the way.

Life, uninterrupted.
DURAGESIC is indicated for patients in chronic pain who require Patients prefer* the DURAGESIC
continuous opioid analgesia and whose pain cannot be managed
by lesser means such as acetaminophen-opioid combinations, patch. DURAGESIC only needs
nonsteroidal analgesics, or p.r.n. dosing with short-acting opioids.
NOTE: Since elderly, cachectic, or debilitated patients may have altered to be applied once every 3 days,
pharmacokinetics due to poor fat stores, muscle wasting, or altered
clearance, they should not be started on DURAGESIC doses higher than maintains consistent serum levels
25 mcg/hr unless they are taking more than 135 mg/day of oral morphine
or equivalent dose of another opioid. over 72 hours,1 and has a favorable
The most common adverse experiences included nausea (23%), vomiting
(22%), somnolence (17%), constipation (14%), diaphoresis (14%), dry side-effect profile. So give your
mouth (13%), confusion (13%), asthenia (12%), anorexia (8%), dizziness
(7%), and nervousness (6%). Many patients were receiving other regimens, patients a life uninterrupted by
including chemotherapy and radiation.
Please see adjacent page for brief summary of full Prescribing Information,
chronic pain.
including Boxed Warning. Reference: 1. Southam MA. Transdermal fentanyl therapy: system design, pharmacokinetics and efficacy.
Anticancer Drugs. 1995;6(suppl 3):29-34.
*Data obtained from 40 patients in clinical trials at a 1-month follow-up evaluation. Previous analgesic regimens
www.duragesic.com included immediate-release oxycodone preparations and short- and long-acting morphine preparations.
Before prescribing, please consult complete prescribing information of which the following is a brief summary. Ambulatory Patients
Strong opioid analgesics impair the mental or physical abilities required for the performance of potentially dangerous tasks such
BECAUSE SERIOUS OR LIFE-THREATENING HYPOVENTILATION COULD OCCUR, DURAGESIC (FENTANYL TRANSDERMAL as driving a car or operating machinery. Patients who have been given DURAGESIC should not drive or operate dangerous
SYSTEM) IS CONTRAINDICATED: machinery unless they are tolerant to the effects of the drug.
In the management of acute or post-operative pain, including use in out-patient surgeries Carcinogenesis, Mutagenesis, and Impairment of Fertility
In the management of mild or intermittent pain responsive to PRN or non-opioid therapy Because long-term animal studies have not been conducted, the potential carcinogenic effects of DURAGESIC are unknown.
In doses exceeding 25 g/h at the initiation of opioid therapy There was no evidence of mutagenicity in the Ames Salmonella typhimurium mutagenicity assay, the primary rat hepatocyte
(See CONTRAINDICATIONS for further information.) unscheduled DNA synthesis assay, the BALB/c-3T3 transformation test, the mouse lymphoma assay, the human lymphocyte
DURAGESIC SHOULD NOT BE ADMINISTERED TO CHILDREN UNDER 12 YEARS OF AGE OR PATIENTS UNDER 18 and CHO chromosomal aberration in-vitro assays, or the in-vivo micronucleus test.
YEARS OF AGE WHO WEIGH LESS THAN 50 KG (110 LBS) EXCEPT IN AN AUTHORIZED INVESTIGATIONAL RESEARCH Pregnancy -- Pregnancy Category C
SETTING. (See PRECAUTIONS - Pediatric Use.) Fentanyl has been shown to impair fertility and to have an embryocidal effect in rats when given in intravenous doses 0.3
DURAGESIC is indicated for treatment of chronic pain (such as that of malignancy) that: times the human dose for a period of 12 days. No evidence of teratogenic effects has been observed after administration of
Cannot be managed by lesser means such as acetaminophen-opioid combinations, non-steroidal analgesics, fentanyl to rats. There are no adequate and well-controlled studies in pregnant women. DURAGESIC should be used during
or PRN dosing with short-acting opioids and pregnancy only if the potential benefit justifies the potential risk to the fetus.
Requires continuous opioid administration. Labor and Delivery
The 50, 75, and 100 g/h dosages should ONLY be used in patients who are already on and are tolerant to opioid therapy. DURAGESIC is not recommended for analgesia during labor and delivery.
Nursing Mothers
INDICATIONS AND USAGE Fentanyl is excreted in human milk; therefore DURAGESIC is not recommended for use in nursing women because of the
DURAGESIC (fentanyl transdermal system) is indicated in the management of chronic pain in patients who require continuous possibility of effects in their infants.
opioid analgesia for pain that cannot be managed by lesser means such as acetaminophen-opioid combinations, non-steroidal Pediatric Use
analgesics, or PRN dosing with short-acting opioids. The safety and efficacy of DURAGESIC in pediatric patients have not been established. (See BOX WARNING and
DURAGESIC should not be used in the management of acute or postoperative pain because serious or life-threatening CONTRAINDICATIONS.)
hypoventilation could result. (See BOX WARNING and CONTRAINDICATIONS.) DURAGESIC SHOULD NOT BE ADMINISTERED TO CHILDREN UNDER 12 YEARS OF AGE OR PATIENTS UNDER 18 YEARS
In patients with chronic pain, it is possible to individually titrate the dose of the transdermal system to minimize the risk of OF AGE WHO WEIGH LESS THAN 50 KG (110 LBS) EXCEPT IN AN AUTHORIZED INVESTIGATIONAL RESEARCH SETTING.
adverse effects while providing analgesia. In properly selected patients, DURAGESIC is a safe and effective alternative to Geriatric Use
other opioid regimens. (See DOSAGE AND ADMINISTRATION.) Information from a pilot study of the pharmacokinetics of IV fentanyl in geriatric patients indicates that the clearance of fentanyl
CONTRAINDICATIONS may be greatly decreased in the population above the age of 60. The relevance of these findings to transdermal fentanyl is
BECAUSE SERIOUS OR LIFE-THREATENING HYPOVENTILATION COULD OCCUR, DURAGESIC (FENTANYL TRANSDERMAL unknown at this time.
SYSTEM) IS CONTRAINDICATED: Since elderly, cachectic, or debilitated patients may have altered pharmacokinetics due to poor fat stores, muscle wasting, or
In the management of acute or post-operative pain, including use in out-patient surgeries because there is no oppor- altered clearance, they should not be started on DURAGESIC doses higher than 25 g/h unless they are already taking more
tunity for proper dose titration (See CLINICAL PHARMACOLOGY and DOSAGE AND ADMINISTRATION), than 135 mg of oral morphine a day or an equivalent dose of another opioid (see DOSAGE AND ADMINISTRATION).
In the management of mild or intermittent pain that can otherwise be managed by lesser means such as Information for Patients
acetaminophen-opioid combinations, non-steriodal analgesics, or PRN dosing with short-acting opioids, and A patient instruction sheet is included in the package of DURAGESIC systems dispensed to the patient.
In doses exceeding 25 g/h at the initiation of opioid therapy because of the need to individualize dosing by titrating Disposal of DURAGESIC
to the desired analgesic effect. DURAGESIC should be kept out of the reach of children. DURAGESIC systems should be folded so that the adhesive side of
DURAGESIC is also contraindicated in patients with known hypersensitivity to fentanyl or adhesives. the system adheres to itself, then the system should be flushed down the toilet immediately upon removal. Patients should
WARNINGS dispose of any systems remaining from a prescription as soon as they are no longer needed. Unused systems should be
DURAGESIC (FENTANYL TRANSDERMAL SYSTEM) SHOULD NOT BE ADMINISTERED TO CHILDREN UNDER 12 YEARS removed from their pouches and flushed down the toilet.
OF AGE OR PATIENTS UNDER 18 YEARS OF AGE WHO WEIGH LESS THAN 50 KG (110 LBS) EXCEPT IN AN AUTHORIZED If the gel from the drug reservoir accidentally contacts the skin, the area should be washed with clear water.
INVESTIGATIONAL RESEARCH SETTING. (See PRECAUTIONS-Pediatric Use.) ADVERSE REACTIONS
PATIENTS WHO HAVE EXPERIENCED ADVERSE EVENTS SHOULD BE MONITORED FOR AT LEAST 12 HOURS AFTER In post-marketing experience, deaths from hypoventilation due to inappropriate use of DURAGESIC (fentanyl transdermal
DURAGESIC REMOVAL SINCE SERUM FENTANYL CONCENTRATIONS DECLINE GRADUALLY AND REACH AN APPROXIMATE system) have been reported. (See BOX WARNING and CONTRAINDICATIONS.)
50% REDUCTION IN SERUM CONCENTRATIONS 17 HOURS AFTER SYSTEM REMOVAL. Pre-marketing Clinical Trial Experience:
DURAGESIC SHOULD BE PRESCRIBED ONLY BY PERSONS KNOWLEDGEABLE IN THE CONTINUOUS ADMINISTRATION The safety of DURAGESIC has been evaluated in 357 postoperative patients and 153 cancer patients for a total of 510
OF POTENT OPIOIDS, IN THE MANAGEMENT OF PATIENTS RECEIVING POTENT OPIOIDS FOR TREATMENT OF PAIN, AND patients. Patients with acute pain used DURAGESIC for 1 to 3 days. The duration of DURAGESIC use varied in cancer
IN THE DETECTION AND MANAGEMENT OF HYPOVENTILATION INCLUDING THE USE OF OPIOID ANTAGONISTS. patients; 56% of patients used DURAGESIC for over 30 days, 28% continued treatment for more than 4 months, and 10%
THE CONCOMITANT USE OF OTHER CENTRAL NERVOUS SYSTEM DEPRESSANTS, INCLUDING OTHER OPIOIDS, SEDATIVES used DURAGESIC for more than 1 year.
OR HYPNOTICS, GENERAL ANESTHETICS, PHENOTHIAZINES, TRANQUILIZERS, SKELETAL MUSCLE RELAXANTS, SEDATING Hypoventilation was the most serious adverse reaction observed in 13 (4%) postoperative patients and in 3 (2%) of the cancer
ANTIHISTAMINES, AND ALCOHOLIC BEVERAGES MAY PRODUCE ADDITIVE DEPRESSANT EFFECTS. HYPOVENTILATION, patients. Hypotension and hypertension were observed in 11 (3%) and 4 (1%) of the opioid-naive patients.
HYPOTENSION AND PROFOUND SEDATION OR COMA MAY OCCUR. WHEN SUCH COMBINED THERAPY IS CONTEM- Various adverse events were reported; a causal relationship to DURAGESIC was not always determined. The frequencies
PLATED, THE DOSE OF ONE OR BOTH AGENTS SHOULD BE REDUCED BY AT LEAST 50%. presented here reflect the actual frequency of each adverse effect in patients who received DURAGESIC. There has been no
ALL PATIENTS SHOULD BE ADVISED TO AVOID EXPOSING THE DURAGESIC APPLICATION SITE TO DIRECT EXTERNAL attempt to correct for a placebo effect, concomitant use of other opioids, or to subtract the frequencies reported by placebo-
HEAT SOURCES, SUCH AS HEATING PADS OR ELECTRIC BLANKETS, HEAT treated patients in controlled trials.
LAMPS, SAUNAS, HOT TUBS, AND HEATED WATER BEDS, ETC., WHILE The following adverse reactions were reported in 153 cancer patients at a fre-
WEARING THE SYSTEM. THERE IS A POTENTIAL FOR TEMPERATURE- quency of 1% or greater; similar reactions were seen in the 357 postoperative
DEPENDENT INCREASES IN FENTANYL RELEASE FROM THE SYSTEM. (See patients studied.
PRECAUTIONS - Patients with Fever/External Heat.) Body as a Whole: abdominal pain*, headache*
PRECAUTIONS Cardiovascular: arrhythmia, chest pain
General Digestive: nausea**, vomiting**, constipation**, dry mouth**, anorexia*,
DURAGESIC (fentanyl transdermal system) doses greater than 25 g/h are too diarrhea*, dyspepsia*, flatulence
high for initiation of therapy in non-opioid-tolerant patients and should not be Nervous: somnolence**, confusion**, asthenia**, dizziness*, nervousness*,
used to begin DURAGESIC therapy in these patients. (See BOX WARNING.) hallucinations*, anxiety*, depression*, euphoria*, tremor, abnormal coordina-
DURAGESIC may impair mental and/or physical ability required for the tion, speech disorder, abnormal thinking, abnormal gait, abnormal dreams,
performance of potentially hazardous tasks (eg, driving, operating machinery). agitation, paresthesia, amnesia, syncope, paranoid reaction
Patients who have been given DURAGESIC should not drive or operate danger- Respiratory: dyspnea*, hypoventilation*, apnea*, hemoptysis, pharyngitis,
ous machinery unless they are tolerant to the side effects of the drug. hiccups
Patients should be instructed to keep both used and unused systems out of Skin and Appendages: sweating**, pruritus*, rash, application site reaction
the reach of children. Used systems should be folded so that the adhesive - erythema, papules, itching, edema
side of the system adheres to itself and flushed down the toilet immediately Urogenital: urinary retention*
upon removal. Patients should be advised to dispose of any systems remaining * Reactions occurring in 3% - 10% of
from a prescription as soon as they are no longer needed. Unused systems should be removed from their pouch and flushed DURAGESIC patients
down the toilet. ** Reactions occurring in 10% or more of DURAGESIC patients
Hypoventilation (Respiratory Depression) The following adverse effects have been reported in less than 1% of the 510 postoperative and cancer patients studied; the
Hypoventilation may occur at any time during the use of DURAGESIC. association between these events and DURAGESIC administration is unknown. This information is listed to serve as alerting
Because significant amounts of fentanyl are absorbed from the skin for 17 hours or more after the system is removed, hypo- information for the physician.
ventilation may persist beyond the removal of DURAGESIC. Consequently, patients with hypoventilation should be carefully Cardiovascular: bradycardia
observed for degree of sedation and their respiratory rate monitored until respiration has stabilized. Digestive: abdominal distention
The use of concomitant CNS active drugs requires special patient care and observation. (See WARNINGS.) Nervous: aphasia, hypertonia, vertigo, stupor, hypotonia, depersonalization, hostility
Chronic Pulmonary Disease Respiratory: stertorous breathing, asthma, respiratory disorder
Because potent opioids can cause hypoventilation, DURAGESIC (fentanyl transdermal system) should be administered with Skin and Appendages, General: exfoliative dermatitis, pustules
caution to patients with pre-existing medical conditions predisposing them to hypoventilation. In such patients, normal analgesic Special Senses: amblyopia
doses of opioids may further decrease respiratory drive to the point of respiratory failure. Urogenital: bladder pain, oliguria, urinary frequency
Head Injuries and Increased Intracranial Pressure Post-Marketing Experience:
DURAGESIC should not be used in patients who may be particularly susceptible to the intracranial effects of CO2 retention The following adverse reactions reported to have been observed in association with the use of DURAGESIC and not reported
such as those with evidence of increased intracranial pressure, impaired consciousness, or coma. Opioids may obscure the in the pre-marketing adverse reactions section above include:
clinical course of patients with head injury. DURAGESIC should be used with caution in patients with brain tumors.
Cardiac Disease Body as a Whole: edema
Fentanyl may produce bradycardia. Fentanyl should be administered with caution to patients with bradyarrhythmias. Cardiovascular: tachycardia
Hepatic or Renal Disease Metabolic and Nutritional: weight loss
At the present time insufficient information exists to make recommendations regarding the use of DURAGESIC in patients Special Senses: blurred vision
with impaired renal or hepatic function. If the drug is used in these patients, it should be used with caution because of the DRUG ABUSE AND DEPENDENCE
hepatic metabolism and renal excretion of fentanyl. Fentanyl is a Schedule II controlled substance and can produce drug dependence similar to that produced by morphine.
Patients with Fever/External Heat DURAGESIC (fentanyl transdermal system) therefore has the potential for abuse. Tolerance, physical and psychological
Based on a pharmacokinetic model, serum fentanyl concentrations could theoretically increase by approximately one third for dependence may develop upon repeated administration of opioids. Iatrogenic addiction following opioid administration is
patients with a body temperature of 40C (104F) due to temperature-dependent increases in fentanyl release from the system relatively rare. Physicians should not let concerns of physical dependence deter them from using adequate amounts of
and increased skin permeability. Therefore, patients wearing DURAGESIC systems who develop fever should be monitored opioids in the management of severe pain when such use is indicated.
for opioid side effects and the DURAGESIC dose should be adjusted if necessary. OVERDOSAGE
ALL PATIENTS SHOULD BE ADVISED TO AVOID EXPOSING THE DURAGESIC APPLICATION SITE TO DIRECT EXTERNAL Clinical Presentation
HEAT SOURCES, SUCH AS HEATING PADS OR ELECTRIC BLANKETS, HEAT LAMPS, SAUNAS , HOT TUBS, AND HEATED The manifestations of fentanyl overdosage are an extension of its pharmacologic actions with the most serious significant
WATER BEDS, ETC., WHILE WEARING THE SYSTEM. THERE IS A POTENTIAL FOR TEMPERATURE-DEPENDENT INCREASES effect being hypoventilation.
IN FENTANYL RELEASE FROM THE SYSTEM. Treatment
Drug Interactions For the management of hypoventilation immediate countermeasures include removing the DURAGESIC (fentanyl transdermal
Central Nervous System Depressants system) system and physically or verbally stimulating the patient. These actions can be followed by administration of a specific
When patients are receiving DURAGESIC, the dose of additional opioids or other CNS depressant drugs (including benzo- narcotic antagonist such as naloxone. The duration of hypoventilation following an overdose may be longer than the effects of the
diazepines) should be reduced by at least 50%. With the concomitant use of CNS depressants, hypotension may occur. narcotic antagonist's action (the half-life of naloxone ranges from 30 to 81 minutes). The interval between IV antagonist doses
Agents Affecting Cytochrome P450 3A4 Isoenzyme System should be carefully chosen because of the possibility of re-narcotization after system removal; repeated administration of naloxone
CYP3A4 Inhibitors: Since the metabolism of fentanyl is mediated by the CYP3A4 isozyme, coadministration of drugs that may be necessary. Reversal of the narcotic effect may result in acute onset of pain and the release of catecholamines.
inhibit CYP3A4 activity may cause decreased clearance of fentanyl. The expected clinical results would be increased or If the clinical situation warrants, ensure a patent airway is established and maintained, administer oxygen and assist or control
prolonged opioid effects. Thus patients coadministered with inhibitors of CYP3A4 such as macrolide antibiotics (e.g., respiration as indicated and use an oropharyngeal airway or endotracheal tube if necessary. Adequate body temperature and
erythromycin), azole antifungal agents (e.g., ketoconazole), and protease inhibitors (e.g., ritanovir) while receiving DURAGESIC fluid intake should be maintained.
should be carefully monitored and dosage adjustment made if warranted. If severe or persistent hypotension occurs, the possibility of hypovolemia should be considered and managed with appropriate
CYP3A4 Inducers: Cytochrome P450 inducers, such as rifampin, carbamazepine, and phenytoin, induce metabolism and as parenteral fluid therapy.
such may cause increased clearance of fentanyl. Caution is advised when administering DURAGESIC to patients receiving Manufactured by: Distributed by: 7500315
these medications and if necessary dose adjustments should be considered. ALZA Corporation, Janssen Pharmaceutica Products, L.P. Revised January 2000, February 2001
Drug or Alcohol Dependence Mountain View, CA 94043 Titusville, NJ 08560 Janssen 2001
Use of DURAGESIC in combination with alcoholic beverages and/or other CNS depressants can result in increased risk to
the patient. DURAGESIC should be used with caution in individuals who have a history of drug or alcohol abuse, especially if
they are outside a medically controlled environment.
R A M I F I C AT I O N S

Trigeminal Neuralgia
Robert F. Saul, MD
Associate Professor of Neurology and Neuro-Ophthalmology
Department of Neurology
Virginia Commonwealth University Health System
MCV Hospitals & Physicians

I am discussing Trigeminal Neuralgia The painful paroxysms are more common and day, deprived her of sleep, and
(TN) in this special edition on pain man- in the maxillary and mandibular divisions forbade her some of the bodily
agement for several reasons. First, al- of the trigeminal nerve. They may be functions necessary for life.4
though most physicians are familiar with triggered by light tactile stimulation (fin-
In fact, these periodic agitations became
the classic entity, tic douloureux, it still ger touch, shower spray, shaving, eating,
so frequent, they allowed only five or six
comes clinically cloaked in enough guis- drinking, or brushing ones teeth), and
minutes of peace during an entire hour;
es to present a diagnostic challenge. Sec- are preceded by a brief latent period fol-
the patient could not drink, eat, cough,
ondly, there are really no better tests lowing the stimulus. The neurological ex-
spit, or wipe her face without renewing
than the history, physical examination, amination is normal, unless signs of CNS
all her pains.5 From this clinical picture,
and neuroimaging to reach a correct diag- dysfunction or cranial neuropathy are
one can easily envision the agony these
nosis. Thirdly, I wish to review the treat- present, as seen in patients with MS,
patients endure.
ment of this disabling entity. skull base tumors, or aneurysmal dilata-
tions of the carotid and vertebral-basilar Abnormal central or peripheral nervous
According to Fromm, quoting Leriche,
arterial systems. system function, in the trigeminal root,
the only tolerable pain is someone elses
has been suggested in the pathogenesis of
pain.1 And, attributed to Proust, we are The earliest description of TN is said to
TN. The primary abnormality also may
willing to procrastinate with all our sen- be from the eleventh century, and inter-
be found in the trigeminal ganglion.
sory perceptions except painpain we estingly, the etiology was attributed to
Compression of the blood vessels induces
obey instantly.2 These quotations offer vascular compression, a concept accepted
ephaptic transmission (or ectopic pulse
an analogy of some of the office encoun- today. Nicholas Andre, in 1756, is credit-
generation) by demyelinating trigeminal
ters between physicians and their patients ed with coining the term tic douloureux,
root sites, or by indirectly altering pain-
with TN. and providing the first comprehensive
inhibiting circuits in the trigeminal nucle-
clinical description.3 His first patient is
By definition, TN is a disorder of the sen- us. Even though the exact pathogenesis of
said to have had several teeth extracted in
sory divisions of the fifth cranial nerve, TN is not known, we can effectively treat
an attempt to treat an infection of the
characterized by brief, excruciating patients pharmacologically and surgically.
maxillae, a scenario still not uncommon
paroxysms of stabbing knife-like, or elec-
even today. In his paper, Andre explained, Voltaires remark of doctors pour drugs,
tric shock-like, bursts of pain in the cuta-
What had been regarded as of which they know little, for diseases of
neous distribution of one or more branch-
the end of a mild and tolerable ail- which they know less, into patients of
es of this nerve. The incidence is four to
ment, became the source of the whom they know nothing, not so long
five per 100,000 population and 1% of
sharpest and most uncomfortable ago would have been quite appropriate to
multiple sclerosis (MS) patients. About
pains, I would say the start of a tic the treatment of TN.6 Phenytoin was just
2-3% of TN patients have MS.
douloureux, that assailed her night discovered in 1938. In 1942, Bergouigo-

Continued on page 24

23
May 2002
R A M I F I C AT I O N S

nan, building on Trousseaus deduction stereotactic radiosurgery employing 60- ed patients, but I have yet to encounter
that paroxysmal trigeminal nerve dis- 90 Gy has been achieving good results in this extreme situation in my practice. In
charges were similar to the paroxysmal some difficult cases.10 the case of drug allergy, baclofen, topira-
cerebral activity in epilepsy, employed mate, and lamotrigine are useful adjuncts.
In over twenty years of treating people
phenytoin as the first truly effective drug After three-to-six months of successful
with facial pain, I rarely see the classic
in the treatment of TN.7 Today, with the therapy, I titrate the drug backwards, to
case of isolated tic douloureux. Many pa-
addition of newer first-line agents, see if a spontaneous remission has oc-
tients have an indolent chronic discom-
phenytoin has taken a back seat. curred. If adequate medication fails, neu-
fort in one or more regions on one side of
rosurgical consultation is advised.
Currently, carbamazepine is probably the their face, which usually adheres to the
most effective pharmacological agent, but appropriate dermatomes. The constant
has potential dose-limiting side effects. discomfort is accentuated by more typical
References:
Gabapentin has been a very useful drug in brief paroxysms of pain. These parox-
1G. Fromm, B.J. Sessle eds. Trigeminal Neuralgia.
my own practice, with low toxicity and a ysms of a burning or electrical sensation Boston: Butterworth-Heineman, 1991.
wide dose range. Baclofen is a good sec- suggest neuralgia, which warrants a trial 2Id.
ond-line drug, which has been in use of anticonvulsants. Always ask about tac-
3Id.
since 1980, and we now have three newer tile triggers and dental pathology.11 One
4Id. at 2.
anticonvulsants, topiramate, oxcarbazine, must ask about a history of facial cancer
5Id. Fromm, supra
and lamotrigine. Misoprostol, a or MS. In my experience, concomitant
prostaglandin E analogue, has shown formications, cutaneous analgesia, 6Id.

promise in treating TN in patients with diplopia, or persistent indolent epistaxis 7Id.


MS.8 One must remember that TN is a re- are harbingers of more invasive or com- 8A.T. Reder, B.G.W. Arnason, Trigeminal
lapsing and remitting disorder, and con- pressive skull-base lesions which require Neuralgia in Multiple Sclerosis relieved by a
prostaglandin E analogue, 45 NEUROLOGY
tinuous long-term pharmacological treat- endoscopic paranasal sinus examination,
1097-1100 (1995).
ment is not warranted without attempts at cranial MRI, and sometimes sinus or
9Id.
drug holidays. skull-base CT.
10D. Kondziolka et al., Stereotactic radiosurgery
If the neurological examination is normal, If there are no confounding factors, I start for the treatment of Trigeminal Neuralgia, 18
CLIN. J. PAIN 42-47 (2002).
neuroimaging is unremarkable, and ade- the patient on gabapentin, 100-300 mg.
11W.P. Cheshire, The Shocking Truth About Trigem-
quate drug therapy fails, surgical options t.i.d., and increase it weekly, to a maxi-
inal Neuralgia. 342 N. ENGL. J. MED.
may be highly beneficial. Percutaneous mal dose of 1200 mg. t.i.d., depending on 2003 (2000).
radiofrequency gangliolysis and rhizoto- the response. If no improvement is seen,
my, retrogasserian glycerol injections, or the pain is unbearable from the begin-
and microvascular decompression are all ning, I use carbamazepine, beginning at
effective. It is theorized that the mi- 200 mg. a day, and increase it every few
crovascular decompression works by re- days to a range of 600-800 mg./day in
activating herpes simplex virus and gan- three divided doses, measuring serum
glion neuron transcription alteration, drug levels, before elevating the dosing
causing a controlled injury of the trigemi- further. Intravenous fosphenytoin has
nal root or ganglion.9 More recently, been used to quickly load severely affect-

24
May 2002
R A M I F I C AT I O N S

Distinguishing Between Chronic Pain


and Drug-Seeking Behavior in Pain
Management/Opioid Abuse
Michael Weaver, MD
Assistant Professor of Internal Medicine and Psychiatry
Division of General Medicine and Primary Care
Medical College of Virginia, Virginia Commonwealth University

Pain is the most common complaint con- diction.9 It will disappear if the patients requestsmore pain medication. Starting
veyed to physicians by patients. Ten to six- pain is controlled adequately. To avoid at the lowest dose, with long intervals be-
teen percent of outpatients seen by general pseudoaddiction, believe that the patients tween analgesic doses, may prolong un-
practitioners and twenty-five to forty per- complaints are legitimate, but continue to necessarily the time within which to
cent of hospitalized patients have prob- observe the patient closely to determine reach effective analgesia. This may
lems related to drug or alcohol addiction.1 whether the present therapy is beneficial. prompt some patients to resort to drug-
No firm conclusions may be drawn about seeking behavior. Observation of individ-
the rate of addiction that results from pa- Some types of drug-seeking behavior may ual patient behavior will aid in distin-
tient abuse of pain-requiring opioids, be- be more predictive of prescription medica- guishing between pseudoaddiction and
cause medical literature is inconsistent tion abuse due to addiction.10 These include addiction. The distinction may not be ob-
concerning the study of patient popula- multiple episodes of lost prescriptions; vious during the first few visits, but will
tions or definitions of addiction.2-8 Drug- multiple dose escalations without prior au- become apparent as the physician learns
seeking behavior may be due to addiction thorization, despite warnings that this is in- more about the patient through subse-
or undertreatment of pain. Despite harmful appropriate; and concurrent alcohol or il- quent visits and adjustment of the opioid
consequences, addicted patients display a licit drug abuse. Evidence of deteriorating dose.
loss of control with continued drug use, work performance and family relation-
ships, or reduction of social activities due A nonaddicted patient experiencing pain
which eventually will become apparent to
to the effects of medication, may indicate will stop escalating the opioid dose once
the observant physician.
consequences of addiction. More flagrant the pain is controlled. Often, patients will
Manifestations of drug-seeking behavior behaviors, such as stealing or borrowing decrease the dose slightly as they become
Drug-seeking behavior may manifest as medication from others, or obtaining pre- comfortable and feel that the physician
aggressive complaints about the need for scription drugs from nonmedical sources will continue adequate dosage of the
more pain medication, or phone calls at (the street), demonstrate a level of des- medication. Patients who use analgesics
night or on weekends when a physician is peration that is more consistent with addic- appropriately for pain control will not
more likely to order a prescription. Pa- tion. Injection of an oral formulation of a show signs of addiction if given adequate
tients may request specific medications by prescription medication is risky behavior doses to control pain. These patients
name, such as Demerol, or use pain med- that indicates loss of control by the patient. focus primarily on side effects with the
ication to treat another symptom, such as These behaviors should prompt the physi- goal of maintaining function. Tolerance
anxiety or insomnia. On occasion, patients cian to immediately address the possibility to the analgesic effects of opioids does
may escalate the dose without a doctors of addiction with the patient. not develop rapidly in patients who are
permission or obtain similar medications medicated adequately for pain;11 however,
from an emergency department or another Differentiating drug-seeking behavior selective tolerance develops quickly to
physician. It is difficult to differentiate an addicted pa- euphoric effects, thereby requiring larger
tient who escalates the dose of medication doses to achieve the same effects. An ad-
Undertreatment of pain may lead to drug- to obtain euphoria from a nonaddicted pa- dicted patient usually will escalate the
seeking behavior by the patient. The mani- tient with undertreated pain, since both ex- opioid dose to try to achieve euphoria.
festation of drug-seeking behavior to hibit drug-seeking behavior. The initial Signs of intoxication may become appar-
achieve pain relief is known as pseudoad- course of action for the physician should
be to provide the patient with that which he Continued on page 26

25
May 2002
R A M I F I C AT I O N S

ent, such as sedation and confusion, but the Dealing with addiction 7D.A. Fishbain et al., Drug abuse, dependence and
patient will continue to request larger doses For less serious addiction problems, it is addiction in chronic pain patients, 8 CLIN. J. PAIN
77-85 (1992).
and focus solely on the dosage rather than appropriate to initiate closer monitoring
8K. Kouyanou et al., Medication misuse, abuse
side effects or consequences. Addicted pa- with more frequent visits and to limit the
and dependence in chronic pain patients, 43
tients often will focus on taking opioids amount of medication prescribed at a time. J. PSYCHOSOMATIC RES. 497-504 (1997).
rather than following recommendations for Referral of the patient for specific addic- 9D.E. Weissman & J.D. Haddox, Opioid
other forms of treatment. These differences tion treatment may be appropriate. In cases pseudoaddiction: an iatrogenic syndrome,
36 CLIN. J. PAIN 363-66 (1989).
in behavior pattern help physicians differ- of health-endangering behavior or crimi-
10R.K. Portenoy, Opioid therapy for chronic
entiate pseudoaddiction from addiction. nal activity, such as prescription forgery or
nonmalignant pain: current status, in 1
diversion of controlled substance medica- PROGRESS IN PAIN RESEARCH AND MANAGEMENT,
Monitoring for addiction
tions, the physician must stop providing PHARMACOLOGICAL APPROACHES TO THE TREATMENT
Most pain problems will not improve for OF CHRONIC PAIN: NEW CONCEPTS AND CRITICAL ISSUES
opioids due to the risk of continued abuse.
patients who abuse alcohol, drugs, or pain (IASP Pub., H.L. Fields & J.C. Liebeskind eds.,
Concerns are not only for the safety of pa- 1994).
medication. Therefore, physicians should
tients with serious medication abuse, but 11R. Melzack, The tragedy of needless pain. 262
make a conscious effort to identify and ad-
also for legal and licensing issues if the SCI. AM. 27-33 (1990).
dress addiction.12 Physicians can actively
physician knowingly prescribes medica- 12S.R. Savage, Addiction in the treatment of pain:
monitor for signs of medication misuse.
tions that are being diverted. A physician significance, recognition, and management,
Count the amount of remaining opioid 8 J. PAIN SYMPTOM MGMT. 265-78 (1993).
should taper the patient from opioids in a
medication at each office visit to determine 13S.H. Schnoll & J. Finch, Medical education for pain
controlled fashion, continue to see the
whether the patient has used all available and addiction: making progress toward answering a
patient, and provide other forms of pain need, 22 J. LAW MED. ETHICS 252-56 (1994).
doses. Obtain random urine drug screens
treatment without controlled substance
for the presence of illicit drugs or controlled
prescriptions.
substances not prescribed by the physician.
Urine screening verifies whether the patient Controlled substances are beneficial for the
is taking the prescribed medication. Infor- good health of many patients. Physicians
mation provided by the patients family or should be comfortable prescribing opioids
friends about the patients behavior outside for pain, since the majority of patients
the physicians office may be addressed di- will not suffer problems associated with
rectly with the patient. Physicians should addiction.
view with suspicion patients who claim that
References
their previous physician has retired or make
1B. KISSEN, Medical management of alcoholic
excuses why previous charts are not avail- patients, TREATMENT AND REHABILITATION OF THE
able. The physician is under no regulatory CHRONIC ALCOHOLIC (Plenum Pub.Co. 1997).
or legal obligation to prescribe on demand 2T. Maruta et al., Drug abuse and dependency in
or at the first visit. This prevents patients patients with chronic pain, 54 MAYO CLINIC PROC.
241-44 (1976).
with a criminal intent from walking into the
3LB. Ready et al., Self-reported vs. actual use of
office and immediately receiving large
medications in chronic pain patients, 12 CLIN. J.
doses of controlled substances. Many phar- PAIN 285-94 (1982).
macists will call physicians to alert them to 4J.A. Turner et al., Drug utilization pattern in
the problem of receiving prescriptions to be chronic pain patients. 12 CLIN. J. PAIN 357-63,
supra.
filled from several sources.13 Document on
5F.P. Buckley et al., Medication management in pa-
the patients chart any inappropriate behav-
tients with chronic nonmalignant pain: a review of
ior regarding pain medications, along with the use of a drug withdrawal protocol, 26 CLIN. J.
a plan for follow-up discussion with the pa- PAIN 153-66 (1986).
tient or closer monitoring. 6R.D. Finlayson et al., Substance dependence and
chronic pain: profile of 50 patients treated in an
alcohol and drug dependence unit, 26 CLIN. J. PAIN
167-74 (1986).

26
May 2002
EXCELLENCE in
healthcare demands
PASSION for
healthcare.

Behind every great hospital, theres a tireless


group of men and women who make the difference
between good care and superior care.
These are the kind of dedicated professionals who
serve every day at HCA Richmond Hospitals.
In fact, we attract some of the finest caregivers
anywhere by offering the clinical and educational
opportunities to help them reach the top of their field.
By giving them the resources they need to excel.
And by providing the technology to deliver exceptional
care thats on the leading edge of medicine.
Were passionate about healthcare. And it shows.

Where passion drives excellence.

CJW Medical Center Henrico Doctors Hospitals John Randolph Medical Center
Retreat Hospital Hanover Outpatient Center Tuckahoe Surgery Center

www.hcarichmond.com
R A M I F I C AT I O N S

Chronic Pain Patient or Drug Seeker


Special Agent Carlton Johnson, Jr.
Pharmaceutical Diversion Investigation Unit
Virginia State Police

I appreciate the opportunity to enlighten cians may assist the State Police in these est patient off guard, and it will be easy
the medical community, from a law-en- cases by asking their patients whether to detect attempted fraud. Most people
forcement viewpoint, to the criminal ac- they are being treated by any other physi- carry some kind of picture identification,
tivities of drug seekers, and hope that we cians, and if so, what medications have and it is wise for you to require adequate
remain partners in effectively lessening, been prescribed. Please note the answer identification before treating a patient.
if not eradicating, this activity. I joined to these questions in the patients file.
The next method of pharmaceutical di-
the Pharmaceutical Diversion Investiga- This is valuable evidence to us when try-
version by drug seekers is prescription
tion Unit of the Department of State Po- ing to prove a case of fraud. Quite often,
forgery. A forgers favorite method is to
lice in 1995, and was surprised to learn a doctor shopper will pay for the visit
steal a physicians prescription pad.
that approximately a third of the drug with cash rather than use health insur-
Once the physician becomes distracted
cases investigated by the Bureau of ance. They often will suggest which med-
or leaves the examination room, the pa-
Criminal Investigations are pharmaceuti- ication works best for them and will
tient will take all or part of a prescription
cal cases. Health care professionals re- claim to be allergic to medications they
pad that is in plain sight. The forger also
port the majority of these cases to the do not prefer.
will search for prescription pads in the
State Police. While most cases involve
Another scam occurs when the patient re- cabinets and drawers of an examination
patients who seek various narcotic pain
ceives a prescription, such as Percocet. room. One drug seeker confessed that,
medications and stimulants illegally,
Later, the patient will report to the physi- after attempting unsuccessfully to scam
some involve health care professionals as
cians office that he or she is experienc- physicians in a hospital emergency room,
suspects, the very people who report
ing a bad reaction to the Percocet and she roamed the hospital halls until she
cases to us.
will request something less strong. The found a physicians office door unlocked.
The Pharmaceutical Diversion Investiga- physician usually will prescribe hy- She searched the office until she found
tion Unit depends on health care profes- drocodone. The drug seeker now has ob- prescription pads. A prescription pad is
sionals to alert us to ongoing criminal ac- tained two prescriptions. We realize this more important than cash to a drug seek-
tivity, and we are grateful for this is a judgment call that only the physician er, so remember to secure those prescrip-
assistance. In the remainder of this arti- can make, but if you note the incident tion pads.
cle, I will provide you with information and the additional prescription in the pa-
Still another scam involves the photo-
concerning drug-seeking activity as expe- tients file, it later may be used as evi-
copying of physician-written prescrip-
rienced by law enforcement and will sug- dence if we discover that the patient is re-
tions. With the technology available
gest ways that we may further assist each ceiving Percocet from other physicians.
today, it is difficult to distinguish a pho-
other in diminishing this activity. The doctor shopper also will provide in-
tocopy from an original. The word
formation about himself or herself that is
I bring to your attention some of the void, however, will appear on the pho-
false, such as name, social security num-
common methods used by drug seekers tocopy of a prescription that has been
ber, date of birth, telephone number, and
to illegally obtain pharmaceuticals. Ill written on prescription pads printed on
address. The physicians office staff may
begin with doctor shopping. The term special paper. Although these pads are
request to see a drivers license to verify
doctor shopping refers to the patient expensive, they ultimately may save the
patient identification, preferably after the
who attempts to obtain medications from physician valuable time.
patient has completed the new patient in-
multiple physicians. Defrauding a physi-
formation form. This will catch a dishon-
cian in this manner is a felony. Physi-

28
May 2002
R A M I F I C AT I O N S

The drug seeker who telephones in a pre- Altering prescriptions is also a favorite I have not covered every method used by
scription to the pharmacy is very com- scam of the drug seeker. The offender drug seekers in this article, nor have I seen
mon. The fraudulent caller will claim to will alter the quantity of prescribed pills, or heard of every scam they may devise. I
be the prescribing physician or a physi- e.g., they will change the number 10 to do believe, however, that if you watch for
cian staff member. Most pharmacists are 100. The physician may counter this by the indicators I have provided and imple-
skilled at detecting fraudulent call-in pre- writing the prescription as one would ment the safeguards, you will deter many
scriptions and will contact the prescribing write a check, by both printing and drug seekers.
physicians office to verify suspicious spelling out the numeral(s). The physi-
Please feel free to contact me if you have
prescriptions. It is important that the cian should also spell out the number of
questions or need assistance in handling a
pharmacist have quick and easy access to refills on the prescription to avoid alter-
pharmaceutical diversion problem. You
the physician or his staff so that a fraudu- ation. Some physicians use duplicate pre-
may reach me at 804-659-2331 (pager), or
lent prescription will not be filled. If a scription pads to help combat this prob-
804-323-2469 (voice mail). The contact
fraudulent prescription is verified in a lem. Duplicate prescription pads are a
person in the western portion of the state is
timely manner, the pharmacist often will good recordkeeping method and may be
William S. Purcell, Assistant Special
contact the local police who may arrest used as evidence in the prosecution of a
Agent in Charge, 540-387-5553 (office).
the offender at the pharmacy. drug seeker.

Listening to the Story of Suffering: Psychiatric


Perspectives of Pain Diagnosis and Management
James E. Sellman, MD
President and Practitioner
James E. Sellman, MD & Associates, PC
Clinical Director Community Education and Integrated Medicine
Poplar Springs Hospital

Chronic pain and psychological disorders ic to pain treatment is the essential need sick role. The U.S. Commission on Eval-
are the two most common problems seen to identify and diagnose psychopatholo- uation of Pain lists six features of chronic
by American physicians. In the United gy prior to embarking on diagnostic and pain syndrome:
States, in any given year, 28% of the treatment procedures. Equally axiomatic
1. Intractable pain greater than six
American population suffer from chronic is the imperative requirement to distin-
months.
pain, and an additional 22% suffer from a guish chronic nonmalignant pain and
diagnosable mental disorder, especially chronic pain syndrome. Chronic nonma- 2. Marked alteration of behavior, depres-
depression, followed by generalized anxi- lignant pain refers to pain that is both sion, or anxiety.
ety disorder, somatization disorder, drug persistent and not associated with pro- 3. Marked restriction of daily activities.
dependence, and, less frequently, person- gressive tissue destruction. Some pa-
ality disorders. Chronic pain is a complex tients with chronic pain develop a pattern 4. Excessive use of medication and med-
biopsychosocial phenomenon in which of behaviors, difficulties, and health-care ical services.
biological, psychological, and social fac- utilization that is best described as 5. No clear relationship to organic
tors dynamically interact. Consequently, chronic pain syndrome. Chronic pain disorder.
two-thirds of all patients diagnosed with syndrome is as much a behavioral prob-
6. Multiple nonproductive tests, treatment,
chronic pain also experience significant lem as one of nociception, and its major
and surgeries.
levels of psychological distress. Axiomat- feature is the excessive adoption of the
Continued on page 30

29
May 2002
R A M I F I C AT I O N S

Patients with chronic pain syndrome fre- physicians, or chance happenings. Those A special concern is the diagnosis of ad-
quently have moderate to severe psy- patients whose locus of control is internal diction in chronic pain patients. The cor-
chopathology. They have a story to tell believe that they, themselves, play a nerstones of addiction are:
you, and its a story of suffering. Suffer- major role in determining their future.
1. Preoccupation with pain and
ing takes many forms, including depres- Such people feel and function better than
medication.
sion, anxiety, somatiform disorders, drug others who see events as controlled by
dependence, pathological operant learn- fate or other people. Chronic pain pa- 2. Loss of control of medication use.
ing, personality disorders, impaired cog- tients with chance external locus of 3. Ignoring adverse consequences of be-
nition, labile affect, unstable interperson- control feel depressed, anxious, and help- havior and medication.
al relationships, and poor impulse control. less to deal with their pain, and rely on
Many of these people are trapped in the maladaptive coping strategies. Decreased Addiction in pain patients should be sus-
regression of the sick role that typifies perceptions of self-control may explain pected when such patients exhibit fre-
chronic pain syndrome, complicated by much of the relationship between pain quent intoxication, irritability, mood
impaired boundaries and few resources. and depression. These people often make changes, and inattention to hygiene, and
statements such as I will resume living the suffering or sick-role behavior seems
Psychological determinants of chronic significantly disproportionate to the sus-
after I am well, I cant go out if Im in
pain may be better understood using the pected medical pathology. It is difficult to
pain, and I shouldnt exercise if it
self-discrepancy theory (SDT) and the distinguish inappropriate behavior due to
hurts. Such a belief system has an obvi-
concept of locus of control. SDT pro- addictive disorders from that caused by
ously negative effect on adaptation and
poses that conflicting or incompatible be- psychological or behavioral factors. Pa-
results in an endless cycle of depression,
liefs about the self will lead to emotional tients should be asked about multiple
tension, anger-blame, and an increasing
discomfort. The discrepancy between the sources of drugs, forged prescriptions,
sense of hopelessness and helplessness.
actual and ideal self focuses on unat- whether their physicians have been con-
By understanding SDT and the concept
tained goals and lack of positive out- cerned about their medication consump-
of locus of control, the clinician can
comes. This leads to frustration, disap- tion in the past, and whether they find it
begin to confront these problems and
pointment, and dissatisfaction, which are necessary to change physicians frequent-
make appropriate multidisciplinary refer-
associated with anger against the self, ly. The familys role in the patients ad-
rals. Claudio, in Act III of Shakespeares
self-blaming, and guilt. In chronic pain, diction may pose special difficulties. My
Measure for Measure, stated: The mis-
we find that distress is a direct result of a experience is that patients and their fami-
erable have no other medicine, but only
discrepancy between actual and ideal lies can be astonished at the reduction in
hope. Listening to the patients story of
selves because of the impact of pain and pain and suffering that occurs after detox-
suffering can be quite time-consuming
illness. Clinically, it is important to sepa- ification. Some of these findings suggest
and often associated allied staff are better
rate patients with chronic pain or distress that continued use of harmful substances
able to do this. It is important to know if
about the situational constraints second- is due more to ignorance than to denial. If
the patients parents experienced chronic
ary to the pain from those who have the the cornerstones of addiction are present,
pain, depression, or alcoholism/drug ad-
additional emotional baggage of believ- then a referral to someone with experi-
diction. Additionally, exploring the pa-
ing that the negative consequences of ence in addiction medicine is medically
tients childhood for physical, sexual or
pain mean that they are fundamentally necessary.
emotional abuse, neglect, abandonment,
flawed and worthless. A second concept
or early responsibility is fundamental to Guidelines for pain management include
is locus of control, which refers to the
understanding their suffering, their story, pharmacotherapy and psychological in-
perception that events are either a conse-
and to providing appropriate diagnosis terventions. Experience and expression of
quence of the individuals own behavior
and treatment. pain are complicated multifactoral
[internal control] or are contingent upon
outside forces, such as family members, events. Most clinicians, however, ignore
these factors and, consequently, miss an
excellent opportunity to achieve more sub-
stantial benefits for their patients.

30
May 2002
R A M I F I C AT I O N S

Major issues must be addressed, including: CBT puts the patient in charge of recov- 6. Provide education and empowerment,
ery rather than the burden being carried which could include improved sleep
1. Fear.
by the physician. The patient is the part- hygiene, doctor/patient communica-
2. Hypervigilance and excessive attention ner in both intervention and prevention. tions, assertiveness skills, and even
to pain sensation. CBT emphasizes realistic expectations biomechanics and anatomy. It is im-
3. Catastrophizing and worrying. and patient-driven goals regarding recov- perative with chronic pain to have a
ery, switching the emphasis from sick- contract that specifically details restric-
4. Avoidance. ness to wellness. CBT is aimed at reduc- tions on phone calls between office
5. Depression. ing or extinguishing the influence of visits, using more than one physician,
factors that maintain the patients malad- and rules regarding lost medications.
6. Anger.
dictive behaviors, including belief sys- My rule is to give everyone a one-time
7. Self-denigration. tems and patterns of thought. There are exception to the contract with dis-
six elements of CBT that may be utilized missal from the practice if subsequent
If unaddressed, all of these factors make
in the physicians office in just a few infractions occur.
not only the patient but also the physician
minutes.
miserable, resulting in frustration and Summary and Conclusion: The experi-
failure for all. Chronic pain patients expe- 1. Address direct positive reinforcement ence of chronic pain and disability may
rience not only pain but also associated of patient behavior by being sensitive be understood only if psychological fac-
problems such as sleep difficulties, fa- to situations in which patients are di- tors are considered central to the delivery
tigue, and depression. They become in- rectly enforced for pain behaviors that of effective treatment. Ultimately, the pa-
creasingly interpersonally isolated and seek to minimize the effect. tient is in charge of his own recovery and
develop dysfunctional family roles and there is no chance of success if a psycho-
2. Address indirect positive reinforce-
responsibilities. Nontherapeutic reliance logical plan, often with multidisciplinary
ment of pain behavior, specifically
on social and medical support systems staff, is not in place. Locus of control
avoidance behavior. CBT benefits pa-
develops with these patients who contin- must be shifted internally. Issues of self-
tients by encouraging them to set
ue to seek and receive treatment. For pa- discrepancy, unrealistic expectations, and
achievable goals and avoid symptom-
tients with complex presentations or the high co-morbidity of other psychi-
contingent patterns of activities, which
chronic pain syndrome, antidepressants, atric illness are keys to effective treat-
ultimately result in a steady decline of
psychotropic anticonvulsants, and some ment. Recognizing that pain is truly suf-
overall activity.
of the new atypical antipsychotics fering and listening to the story the
(Risperdal, Zyprexa, Geodon, Seroquel) 3. Provide positive reinforcement of well patient has to tell are critical in achieving
often used in combination can be enor- behavior. any advances. As Dr. Jerri Nielson in her
mously beneficial. Although this is con- 4. Encourage physical fitness and recent book, Icebound, notes, There is
troversial, many clinicians advocate opi- function. no such thing as a crazy patient, just a
ate use in chronic pain and oppose its use stupid doctor. Improvement in pain man-
in chronic pain syndrome. 5. Employ cognitive reframing in which agement may be accomplished by very
self-defeating and self-denigrating pat- simple, if subtle, changes in clinical prac-
Adjunctively, the treatment of choice is terns of thinking are challenged and an tice. Although some of the ideas set forth
cognitive behavioral therapy (CBT), informed psychological perspective in this article are simple, these changes
preferably delivered by an interdiscipli- developed in which one can under- may have significant effects in the pa-
nary pain management team. Without stand the effects of thoughts upon feel- tients experience of pain, ultimate out-
such input, treatments are not only largely ings, including improved problem- come, and his use of health-care
ineffective, but also, at times, even harm- solving, communication, stress services and resources.
ful. Using CBT, the focus of treatment reduction, anger management, and re-
can be shifted away from the pain to- laxation response. The mind is in its own place, and in it-
wards the detrimental effects of pain. self can make a heaven of hell, a hell of
heavenJohn Milton

31
May 2002
R A M I F I C AT I O N S

The Legal Issues: How the Courts See


Pain Management
Stephen D. Rosenthal, Esquire
Troutman Sanders Mays &Valentine, LLP

Based on recent news coverage, you the United States, in which the Court stances, and those laws can provide the
might think that the only legal issues in ruled that physicians registered under the basis for prosecutions and convictions of
pain management today involve the im- federal Controlled Substances Act2 can physicians.6 Thus, many courts appear to
proper prescription of OxyContin and be prosecuted under [the Act] when their view physicians who prescribe pain med-
similar drugs. The unlawful prescription activities fall outside the usual course of ications without legitimate medical rea-
of excessive amounts of pain medication, professional practice.3 Each time a sons as drug pushers rather than pain
however, is not the only source of legal physician knowingly or intentionally pre- managers.7
concern in the field of pain management. scribes a controlled substance without a
Furthermore, courts and juries may hold
Indeed, the underprescription of pain re- legitimate medical purpose, the physician
physicians responsible when their pa-
lief drugs also could cause legal prob- can be charged with unlawfully distrib-
tients overdose from unlawfully pre-
lems for physicians. Although the law of uting a controlled substance.4 As a re-
scribed pain medications.8 A Florida jury
pain management is still developing, the sult, physicians who abuse the pain man-
recently convicted a physician of
cases thus far have shown that physicians agement process over an extended period
manslaughter through culpable negli-
who engage in either improper or inade- of time may face indictments with hun-
gence after four of his patients died from
quate pain management practices run the dreds of counts that carry the potential
overdoses of OxyContin and other drugs
risk of incurring civil liability or criminal for long prison terms under the sentenc-
that he had prescribed. The physician ar-
sanctions. ing guidelines used by the federal courts.
gued during the trial that he had followed
Criminal Consequences for Improper and Last year, for instance, a federal jury con- standard medical practice and could not
Excessive Prescriptions victed a Southwest Virginia physician of be held accountable for patients misuse
Courts appear to view the improper or 427 counts of unlawful distribution of of the drugs that he prescribed. Prosecu-
excessive prescription of pain relief drugs controlled substances for having pre- tors, however, portrayed the physicians
primarily as a criminal concern. Because scribed, among other drugs, pain medica- practice as a prescription mill where
many pain medications are controlled tions such as OxyContin, Tylox, Lorcet, patients loitered outside the office and
substances under federal and state laws, and Lortab without a legitimate medical congratulated one another when they
physicians who improperly prescribe purpose. Under the federal sentencing emerged with prescriptions. The state
these drugs risk criminal indictment for guidelines, the weight of the tablets pre- contended that the physician recklessly
unlawful distribution. In Southwest Vir- scribed, along with other factors, resulted prescribed OxyContin and other narcotics
ginia, for example, a large-scale investi- in a sentencing range of 210-to-262 to anyone willing to pay for an office
gation by state and federal officials into months, although that range was reduced visit.9 In a similar case, California prose-
health care fraud and abuse has led to the by the court to 70-to-87 months because cutors charged the top OxyContin pre-
prosecution of several physicians under of the physicians diminished capacity at scriber under Californias Medi-Cal in-
federal drug laws.1 the time of the offenses.5 surance program with manslaughter in
connection with overdose deaths related
Such prosecutions are made possible by a Many state statutes also prohibit unlawful
to allegedly improper prescriptions.10
1975 decision of the Supreme Court of selling or distribution of controlled sub-

32
May 2002
R A M I F I C AT I O N S

In addition, a physician in Florida awaits during the case focused on the proper liability on physicians in the context of
trial on first-degree murder charges in standard of care in the administration of both under and overprescription of pain
connection with a patients death by over- medications, such as morphine, for pa- medication. Physicians must be aware of
dose. Florida prosecutors based the tients with intractable pain.13 both sides of this legal spectrum when
charge on the states felony murder law, managing their patients pain.
In a more recent case, a California jury
alleging that the patients overdose death
awarded $1.5 million to the family of a
was caused in the commission of felony References
deceased patient on the novel theory that
drug trafficking by the physician. If con- 1News Release, United States Attorneys Office,
the patients physician violated the states
victed, the Florida physician could face Western District of Virginia (Oct.16, 2000), avail-
elder abuse law by providing the patient
the death penalty.11 able at http://www.oig.dol.gov/public/media/oi/
with an inadequate amount of pain med- sutherland.html.
While many of these criminal prosecu- ication. The California verdict is signifi- 221 U.S.C. ch. 13, 801-971 (2000).
tions are a result of particularly egregious cant, because the jury apparently accept- 3United States v. Moore, 423 U.S. 122, 124
behavior by a few physicians, all doctors ed the familys theory that the physician (1975); see also Sharon B. Roberts, All Pushers
who practice pain management should violated the standard of care by underpre- Are Not Created Equal! The Inequities of Sanctions
for Physicians Who Inappropriately Prescribe
be aware of these new theories of scribing medication for his patients pain, Controlled Substances, 23 NOVA L. REV. 881, 886-
prosecution. thus possibly paving the way for future 87 (1999) (discussing Moore).
malpractice claims.14 4Roberts, supra, at 884.
An Emerging Civil Liability Theory for
5United States v. Sutherland, Nos. 1:00CR00052,
Inadequate Pain Management The trend toward recognizing a malprac-
1:01CR00009, 2001 U.S. Dist. LEXIS 19310, at
Although many of the cases involving tice claim, based on a physicians failure *2, *30-31 (W.D. Va. Nov. 27, 2001); see also
pain management are criminal in nature to prescribe an appropriate amount of supra note 1 (discussing drugs prescribed by Dr.
Sutherland).
and stem from the improper and exces- medication to a patient with severe pain,
6Scott W. Parker, Note, An Argument for Preserv-
sive prescription of pain medications, also could draw support from two state
ing the Agency Defense as Applied to Prosecutions
there appears to be a trend developing in supreme court cases dealing with the for Unlawful Sale, Delivery, and Possession of
civil courts that focuses on physicians right to refuse medical treatment. The Drugs, 66 FORDHAM L. REV. 2649, 2664 (1998)
(noting that almost every state legislature [has]
who provide inadequate pain manage- Georgia Supreme Court has stated, and
adopted the Controlled Substances Act into its own
ment by failing to prescribe enough pain the Nevada Supreme Court has agreed, statutory regime).
medication. While there is not yet any de- that the patients right to be free from 7E.g., Moore, 423 U.S. at 138; see also Roberts,
finitive case law establishing that mis- pain, after the patient has made the deci- supra note 3, at 887 (discussing Moore).
management of pain through undermed- sion to refuse treatment, is inseparable 8Barry Meier, OxyContin Prescribers Face Charges
ication is malpractice, several legal from his right to refuse medical treat- in Fatal Overdoses, N.Y. TIMES, Jan.19, 2002,
available at http://www.nytimes.com/2002/
scholars have advocated for such a theory ment in the first place.15 Legal scholars 01/19/health/19OXYC.html.
of liability and have pointed to cases that have contended that the Georgia case in- 9Barry Meier, Doctor Guilty in 4 Deaths
may indicate the direction in which the dicates a recognition by the courts that Tied to OxyContin, N.Y. TIMES, Feb. 20, 2002,
law is moving.12 proper pain management is an integral available at http://www.nytimes.com/2002/02/20/
health/20OXYC.html; Associated Press, Closing
part of medical treatment.16 Arguments Made in Trial of Doctor in OxyContin
For instance, in a negligence lawsuit filed
Deaths, N.Y. TIMES, Feb. 19, 2002, available at
in the early 1990s, a North Carolina jury Thus, although the development of a mal- http://www.nytimes.com/2002/02/19/national/19O
awarded $15 million to the family of a practice theory based on inadequate pain XYC.html.
cancer patient whose last days were made management is still in the early stages, it 10Meier, supra note 8.
more painful by a nurses act of withhold- appears that more courts could be willing 11Peter Franceschina, Saleswoman Testifies in Drug
ing or reducing the amount of pain med- to recognize such claims in the future. Case; Doctor Accused in OxyContin Patients
Death, SUN-SENTINEL (Fort Lauderdale, FL), Jan.
ication that had been prescribed by his Legal scholars point to the promulgation
11, 2002, at 1B; Mitch Lipka & Nancy McVicar,
physician. The nurse determined that the of clinical pain management guidelines OxyContin Prescription Leads to Murder Charge,
patient was addicted to the morphine that as evidence of an emerging standard of CHI. TRIB., July 29, 2001, at C8; Meier, supra
note 8; John Pacenti, Prosecutor: Documents
had been prescribed for him and thus re- care in pain management.17 Tip of Iceberg in OxyContin Case, PALM
duced or withheld the pain medication in BEACH POST, Aug. 30, 2001, at 1B.
In conclusion, courts today seem increas-
favor of mild tranquilizers. Testimony
ingly willing to impose criminal and civil

33
May 2002
R A M I F I C AT I O N S

12Tonya Eippert, Note, A Proposal to Recognize a


Legal Obligation on Physicians to Provide Ade-
Issues Regarding Licensure
quate Medication to Alleviate Pain, 12 J.L. &
HEALTH 381, 382 (1997/1998); Barry R. Furrow,
William L. Harp, MD
Pain Management and Provider Liability: No Executive Director
More Excuses, 29 J.L. MED. & ETHICS 28, 31- Virginia Board of Medicine
34, 43 (2001); Michael J. Reynolds, Note, Mor-
phine or Malpractice: Should Courts Recognize
a Legal Duty to Prescribe Opiates For Treating
Chronic Pain?, 15 ST. JOHNS J. LEGAL COM-
MENT. 79, 81 (2000); Ben A. Rich, A Prescrip-
tion For Pain: The Emerging Standard of Care
for Pain Management, 26 WM. MITCHELL L.
REV. 1, 2-3 (2000); Robyn S. Shapiro, Health
Care Providers Liability Exposure For Inappro- Since 1884, the Board of Medicine has The authority to prescribe medication is
priate Pain Management, 24 J.L. MED. & been issuing licenses and regulating the conferred with the license to practice med-
ETHICS 360-64 (1996).
practice of medicine in the Common- icine. Physicians must be aware of and fol-
13Reynolds, supra note 12, at 105; Rich, supra
wealth of Virginia. As licensed physicians, low the laws and regulations that come
note 12, at 84-86; Shapiro, supra note 12.
we are expected to know and abide by the with that privilege. The Drug Control Act
14Mark Crane, Treating Pain: Damned If You
laws and regulations governing our prac- and related laws are also found in Title
Dont?, MED. ECON., Nov. 19, 2001, at 78; see
also Rich, supra note 12, at 84 (discussing Dr. tice, and physicians who do not may come 54.1 of the Code of Virginia. Specifically,
Chins case). 54.1-3303, 54.1-3404, 54.1-3408 and
to the attention of the Board through the
15State v. McAfee, 385 S.E.2d 651, 652 (Ga. 54.1-3408.1 speak to the lawful use of
complaint process.
1989), quoted in McKay v. Bergstedt, 801 P.2d
controlled substances, including the pre-
617, 631 (Nev. 1990). A word about the disciplinary process at
16Furrow, supra note 12, at 33 n.87; Shapiro,
requisite bona fide physician-patient rela-
the Board is in order. The disciplinary
supra note 12. tionship. Physicians should also be famil-
process is complaint-driven. The Board
17Eippert, supra note 12, at 393-94; Furrow, supra iar with Drug Enforcement Administration
does not have the resources to visit all
note 12, at 31-33; Reynolds, supra note 12, at guidelines and possess a valid DEA num-
95-98, 108; Rich, supra note 12, at 81-83; physician offices and assess all practices.
ber to prescribe Schedules II through V
Shapiro, supra note 12. If, however, a patient or a patients family
drugs.
member, colleague, nurse, pharmacist,
hospital CEO, managed care plan, mal- No one wants to appear before the Board
practice carrier, federal agency, etc., report of Medicine. Such appearance should be
to the Board that a physician may have en- easy to avoid provided one follows the
gaged in unprofessional conduct, then a laws and regulations. But what if the laws
case is docketed and a proper investigation and regulations are difficult for the practi-
ensues. The Board carefully reviews a tioner to interpret?
completed investigative report to deter- It is just a reality that the laws and regula-
mine if probable cause of a violation ex- tions always seem to lag behind the chang-
ists. If the evidence appears likely to sup- ing patterns of clinical practice. New inno-
port a violation of any law or regulation, vations and treatments can become fairly
the physician receives a notice to appear commonplace before it is determined how
before the Board. Unprofessional to address them by law and regulation. In
conduct is defined under the laws that this circumstance, practicing good medi-
govern the Board of Medicine. Sections cine can clash with the laws and regula-
54.1-2914 and 54.1-2915 outline the pro- tions or, at least, appear to do so. In the
scribed behaviors; to be found in violation case of chronic pain, new approaches, at
of one or more may result in a sanction times, have been tested against the Boards
from the Board. interpretation of the laws and regulations

34
May 2002
R A M I F I C AT I O N S

governing both our practice and the pre- ments should result in quality care that is against whom a complaint is filed with the
scription of controlled substances. Physi- well-documented and defensible. Board of Medicine should take heed.
cians who treat chronic pain to the stan- There may be a hearing, and if a violation
Complaints regarding the treatment of
dard of care would like to do so without is found, there may be a sanction.
chronic pain may derive from several
fear that the Board of Medicine may seek The bottom line with the treatment of
sources and contain divergent allegations.
to take their license. chronic pain is to make sure you dot all the
A patient may complain of being under-
In 1997, the Medical Society of Virginia, treated. A family member may feel that Is and cross all the Ts. Stay within the
in conjunction with the Board of Medi- the patient was overtreated. A pharmacist laws and regulations and follow the
cine, developed guidelines for the treat- may report massive amounts of controlled Boards guidelines. Document in a manner
ment of chronic nonmalignant pain. The substances being written for a patient. A for anyone to follow the progression of the
General Assembly allowed the Board of managed care plan may detect a pattern of case and describe your clinical rationale
Medicine to endorse the guidelines, which care that raises concerns. for the approach chosen.
were published in the Board Briefs in The laws, regulations, and guidelines ref-
It is not about the quantity of drug pre-
Spring 1998. They remain an excellent erenced in this article are available for re-
scribed. Quantity alone will not result in a
guide for those who wish to treat chronic view on the Department of Health Profes-
violation if the standard of care is met, and
pain patients and meet the expectations of sions website, at www.dhp.state.va.us.
there is good documentation. As with the
the Board of Medicine in the process. You may access the laws, regulations and
treatment of other clinical conditions, the
Broadly speaking, the guidelines address guidelines for each specific Board by se-
Board is interested in protecting the public
history and physical examination, diagno- lecting from the Boards dropdown menu.
and ensuring that the public receives the
sis, treatment plan, informed consent, opi- You may also search the Code of Virginia
care for which they are paying. Any physi-
oid contract, periodic review, consultation, and the Administrative Code under the
cian who does not meet the standard of
and documentation. Attention to these ele- Law & Policy dropdown menu.
care, who places patients at risk, and

REMINDER
When your malpractice insurance policy renews, please add
CCVS as a certificate holder. We will then automatically receive
a copy of your certificate when it renews and will forward to the
hospitals. Thank you! Our information is as follows:
CCVS, Inc.
1200 E. Clay Street
Richmond, VA 23219

35
May 2002
R A M I F I C AT I O N S

Alternative Therapy Approaches: How


Accupuncture Can Be Effectively Used
M. Craig Pinsker, MD, PhD
Chief of Anesthesia
Richmond Eye and Ear Hospital

If a reliable method of controlling pain ered, it will need to explain not only as a treatment for low back pain,2
were available, there would be no need acupunctures analgesic properties but headaches,3 pain from bony metastasis,4
for this special edition of RAMifications. also its effect on the immune system, its and diabetic neuropathy.5 In the treatment
We could simply prescribe a remedy that antiemetic effect, and its role in control of of pain, it may be more palatable to say
would relieve the pain without side ef- substance abuse, especially cocaine. What that we insert needles into the
fects. Of course, this magical medication is special about acupuncture? paraspinous muscles, not the gallbladder
would not interfere with our normal re- meridian, and that we use PENS to stim-
Acupuncture generally is known as a
sponse to injury, such as stepping on a ulate the needles, not an electroacupunc-
mysterious Eastern technique complete
nail or closing a door on a finger. Since toscope. Hopefully, this will help bring
with well-described theories, nomencla-
this magical formula does not exist, we this procedure into standard medical
ture, and applications that do not fit into
employ polypharmacy to maximize anal- practice. We should consider acupuncture
our Western understanding of nature. Not
gesia and minimize ill effects. We pre- by whatever name we give it as a com-
understanding this science, we tend to
scribe opiates, nonsteroidals, antidepres- plementary, not an alternative, compo-
discount it. It is beyond the scope of this
sants, anticonvulsants, clonidine, nent of standard medical practice. Unfor-
article, and unnecessary at this time, to
ketamine, dextromethorphan, etc. Some- tunately, there is a shortage of physicians
discuss its mechanism of action. We
times we try to block pain fibers with qualified in acupuncture in Virginia. Non-
should recall that, not long ago, we used
local anesthetics or destroy them with medical doctors have filled this gap. I
aspirin without understanding the manner
freezing or burning. Ice, heat, and other feel strongly that patients treated by non-
in which it works. We should note that
physical methods also are used in con- physicians should be monitored by physi-
our new understanding of aspirins mech-
junction with medications. cians so that the acupuncture is a comple-
anism of action does not make it work
mentary, and not an alternative,
Acupuncture, used alone or with other any better! Acupuncture can be very help-
treatment. For physicians treating pa-
modalities, is a physical method of reduc- ful in the treatment of myofascial pain
tients, it is best to regard acupuncture as a
ing pain. Acupuncture (sometimes a syn- and headaches, and it is especially useful
technique that may be used to reduce de-
onym for traditional Chinese medicine) is if nausea is also present. Radicular pain
pendence on medications or other anal-
the insertion of needles to achieve a clini- usually is helped, possibly due to the my-
gesic modalities, thereby producing a
cal response (i.e., analgesia). Acupunc- ofascial aspect of this pain. It is less like-
more comfortable patient.
ture is a form of complementary medi- ly to help with central pain. We ought to
cine. In1997, over 41% of the population try acupuncture on a given patient just as References
in the United States reported using com- we might try any other modality. If it 1D.M. Eisenberg et al., 280 JAMA 1,569 (1998).

plementary medicine.1 Insertion of a nee- helps, continue its use; if not, stop the 2E.A. Ghoname et al., 281 JAMA 818 (1999).
dle causes a current of injury. Could this treatment. We would evaluate any other 3H.E. Ahmed et al., 40 HEADACHE 311 (2000).
be what makes it work? Adding electrical medical treatment this same way. 4H.E. Ahmed et al., 14 CLIN. J. PAIN 320 (1998).
stimulation to the needles adds to the al-
There is an effort under way to create a 5M.A. Hamza et al., 23 DIABETES CARE 365 (2000).
teration of electrical and magnetic fields
new CPT code for percutaneous electrical
in the body. Acupuncture points have de-
nerve stimulation (PENS). Several arti-
creased electrical resistance. Whatever
cles from Paul Whites group described it
mechanism of action is eventually discov-

36
May 2002
Pain, pain, go away...
Bon Secours St. Marys Hospital
Ranked in the Top 10% of Americas Hospitals *
Known for quality care, St. Marys offers a wide range of specialized services, including our
New Life Center, chosen as the best place in Richmond to have a baby,** and our neonatal
intensive care unit. Other acute care services include heart surgery, cardiology, urology, pedi-
atrics, neurology, behavioral health, orthopaedics, cancer care, emergency and outpatient care.

Bon Secours Richmond Community Hospital


A Top Performer in Emergency Room Services+
Recognized for our responsive Emergency Room Services, we offer comprehensive medical,
surgical and behavioral health services in our modern facility. Quality care. Quality doctors.
Quality outreach programs that reach out to our neighbors. At Richmond Community Hospital,
were working to promote wellness in the Richmond community.

Bon Secours Memorial Regional Medical Center


A Top Performer for Stroke Care and Cardiac Services
If youre near Hanover County, then Memorial Regional Medical Center is your neighbor.
Were among the best in the nation in care for stroke and heart patients. This facility offers
a comprehensive range of services including obstetrics/gynecology, neonatal intensive care,
orthopaedics, oncology and 24-hour emergency room care.

Bon Secours St. Francis Medical Center


Our Commitment South of the James
Southern Chesterfield County will have a medical center to call its own in 2003 and will be
the newest addition to the Bon Secours Richmond health care family. St. Francis is
designed to provide affordable and accessible medical care to residents of Chesterfield,
Powhatan, Amelia, Cumberland, Goochland, Buckingham and Nottoway counties, and to
the city of Colonial Heights.

Bon Secours Health System

* Professional Research Consultants survey based on overall + Named a Top Performer in a Professional Research Recognized by HCIA as a Top 100 Performer in stroke care
quality of care and overall quality of doctors care in the ER. Consultants survey based on emergency room patient satisfaction. among hospitals nationwide in 2000 and in cardiac care by
** Style Magazine, 2001 www.healthgrades.com in 2002.
R A M I F I C AT I O N S

The Role of the Physiatrist in Managing


Chronic Nonmalignant Pain
Jane Pendleton Wootton, MD
Physiatrist
Sheltering Arms Physicians Rehab Services

Physiatrists (specialists in physical medi- tient, especially when the patient is threat- measures are unnecessary in todays med-
cine and rehabilitation) have a great deal to ened by sudden termination of insurance ical environment.
offer patients suffering chronic nonmalig- benefits or disability payments. An effec-
The care of an attentive physician within
nant pain. Residency training programs in- tive attorney may lessen the stress and de-
the boundaries of a contract, which clarifies
corporate and emphasize the treatment of spair which aggravate the patients misery.
the patients compliance responsibilities,
chronic pain in patientsa vastly under- Other valuable team members may include offers the patient assurance and hope that
served population. Many of the best pro- massage therapists, pharmacists, pharma- there is a constructive, multidisciplinary
grams, including that of the Medical Col- cologists, acupuncturists, athletic trainers, plan aimed at achieving the most comfort-
lege of Virginia, offer fellowships in ministers, and, of course, nurses who able life possible. All necessary consulta-
chronic pain management. teach, support, counsel, and soothe patients tions and diagnostic procedures should be
The rehab physician offers conservative with their kindness. performed in the early stages of the patient-
(nonsurgical) management of pain, and The physiatrist is responsible for discern- physician relationship, to ensure that the
works as a team player with surgeons and ing whether and when such services are etiology of the pain is delineated clearly,
anesthesiologists who offer palliative or appropriate in a particular case, and for co- insofar as possible, and that no reversible
curative measures, such as neurosurgical ordinating the overall treatment effort. This contributing etiology is missed. Patients
(intracranial or spinal) procedures, amputa- work is quite time and labor intensive, and their families need to know that no
tions, nerve blocks, spinal pumps for deliv- since the treating doctor must follow the stone has been left unturned in uncovering
ery of intrathecal opioids, etc. patient closely and be available by phone, fixable causes of pain.

Family physicians are a most important fax, or e-mail, especially when opioid anal- Far from being the exception, treatment of
part of the team, as are patients families gesics are part of the treatment. Neverthe- chronic nonmalignant pain with opioid
and, most especially, the patient. Clear, less, the opportunity to relieve pain and analgesics is the standard of care today.
honest communication, using plain lan- help the patient move toward a fuller, more This requires courage, strict control over
guage and visual aids, enhances mutual un- fulfilling and purposeful life (whether or the dispensation of opioid medication,
derstanding among the team members. not that includes returning to work) is most close patient follow-up, and a thorough un-
rewarding to the physician. derstanding of not only the patients med-
Rehab physicians specializing in pain man-
It must be understood that chronic pain ical condition, but also his or her reliability,
agement offer the advantage of emphasiz-
(meaning pain that persists longer than six stability, personality traits, support system,
ing a team approach with other highly
months) is a serious, debilitating disruption and environment. It also requires a thor-
skilled health professionals, such as physi-
to a patients life and (usually) work. Most ough knowledge of Virginias laws and the
cal and occupational therapists, psycholo-
of these patients rate the pain at least 3 to 4 need for extensive documentation. Patients
gists, psychiatrists, and social workers.
on a visual analogue scale of 10, with with a history of addiction, borderline per-
Many of these professionals are proficient
10/10 being unbearable. Prolonged seri- sonality disorders, serious mental illness,
at counseling and coaching patients and
ous pain associated with depression leads impaired intelligence, and those in a delin-
families suffering the stress of chronic pain.
many people to consider suicide, or to quent or potentially criminal environment
The services of a dedicated attorney are
abuse alcohol or illegal drugs in their des- usually make poor candidates for such care.
sometimes essential to keep the system
moving in ways that are helpful to the pa- perate search for relief. Such destructive

38
May 2002
R A M I F I C AT I O N S

Publications as extensive as this special edition require considerable underwriting.


The Richmond Academy of Medicine extends its grateful appreciation to the
following companies for providing grant funding of this issue.

Address correction requested

1200 East Clay Street, Richmond, Virginia 23219

Help Your Academy


Save Money!
A fax to your office is $.05 an email is FREE!

If youre not getting emails from us, we dont have


your address or we have an incorrect address for
you!

Please provide the Academy with your email address!

Fax this back to 788-9987


Call Lara at 643-6631 and give her your address
Or, mail this to
1200 East Clay Street, Richmond, VA 23219
PERMIT NO. 1459

ORGANIZATION
RICHMOND, VA

U.S. POSTAGE

NONPROFIT
PAID

Name: ___________________________________

Email Address: ____________________________

Thank you!

You might also like